¿Qué miden los relojes?

Hace poco hice una pregunta sobre la medición del tiempo, y se ha hecho evidente que en realidad estoy haciendo un conjunto de preguntas relacionadas, cuyas premisas deben reforzarse y articularse claramente, particularmente en el contexto de los procesos en la física nuclear. .

Parece que estas preguntas no se pueden formular sin la máxima precisión.

Centrándome en un solo aspecto del problema por ahora, afirmé que los relojes miden la velocidad de un proceso físico que tiene lugar en el espacio así como el "tiempo", en lugar de medir el "tiempo" directa e independientemente. Adaptando el giro de frase adecuado de otra persona con el que me he tropezado, un reloj no es como un medidor de gas a través del cual fluyen cantidades de tiempo puro.

Supongo que esta es una afirmación indiscutible, que los relojes miden físicamente tasas , de una cosa u otra.

Estamos acostumbrados a la idea de relojes que miden mal el tiempo, aquellos que miden el tiempo a un ritmo variable, al ritmo "incorrecto", o incluso dejan de funcionar. Cuando esto ocurre, simplemente los consideramos "relojes defectuosos": no consideramos que el "paso del tiempo" haya sido impugnado por los caprichos de cómo funciona un reloj en particular.

Sin embargo, con los relojes atómicos, parece haber una creencia generalizada de que la "velocidad" del reloj es físicamente constante e invariable.

Sin duda, la tasa de conversión entre el estándar de cesio y el "segundo" se define como constante, pero eso no establece la constancia física del proceso que mide el reloj atómico. Más bien, simplemente afirma que cualquiera que sea la variabilidad en la tasa, también esa variabilidad se reflejará en la definición de cada "segundo".

Este era el problema con los dueños de los molinos del siglo XIX, quienes definían un "día completo" como un período determinado en la esfera del reloj, y controlando el reloj, manipulando su funcionamiento y afectando su velocidad, podían extender la cantidad de tiempo . tiempo que en realidad estaba contenido en un "día completo" en la planta.

Hay, por supuesto, una categoría de resultados experimentales que son capaces de establecer el hecho de que el estándar de cesio es físicamente variable. Todos los experimentos realizados en relatividad, por ejemplo, donde los relojes atómicos van de viaje o experimentan un cambio en su entorno gravitacional, y regresan con lecturas diferentes. Dudo que necesite ensayar los nombres de experimentos específicos para alguien aquí.

En los comentarios de mi pregunta anterior, me referí a cómo en el siglo XIX, los físicos estaban acostumbrados a la idea de que la oscilación de un péndulo no era una constante física. La temperatura, la fricción, el "acoplamiento resonante", todo tipo de variables físicas podrían afectarlo, y hubo una serie de innovaciones diseñadas para corregirlas. Pero nunca perdieron de vista que estas variables estaban presentes, y si la frecuencia del reloj variaba, era por efectos de alguna u otra variable física, y no por algún cambio en el "paso del tiempo".

Pero observo que en la física de la relatividad hoy en día, los físicos no parecen hablar de "relojes que se ralentizan" (o, de hecho, se aceleran, como pueden hacer). En cambio, hablan de " el tiempo se ralentiza".

En otras palabras, ha habido un cambio en el razonamiento de la época clásica, en la que se esperaba que los relojes estuvieran sujetos a variables físicas, a la concepción moderna en la que se considera que la velocidad de un proceso atómico es una constante física. Dado que la velocidad se mantiene constante, si la lectura del reloj varía, se puede atribuir a un cambio en "el paso del tiempo mismo", en lugar de un mero cambio en la velocidad del proceso que se está midiendo.

¿Por qué ha habido este cambio en el razonamiento y qué evidencia experimental justifica tratar la velocidad de cualquier proceso físico como constante? ¿O no está justificado experimentalmente, sino que simplemente representa una diferencia de gusto (en comparación con los físicos clásicos) en cómo se interpreta la medición de un reloj?

Los comentarios no son para una discusión extensa; esta conversación se ha movido a chat .
Hay una diferencia entre "Time" y los mejores relojes que hemos podido hacer. El tiempo (con mayúsculas) por lo que podemos observar parece ser constante, inalterable y solo yendo en una dirección. Los relojes son imperfectos. Hay teorías que parecen decir que el tiempo ha cambiado (por ejemplo, en el Big Bang) y podría estar yendo en ambas direcciones. Interesante pero aún no probado.
@ghellquist, toda la premisa de mi argumento anterior es que no observas el tiempo. Nadie ha observado jamás el tiempo puro. Los relojes no miden el tiempo; miden partes móviles (es decir, partes cuya configuración en el espacio debe verse que cambia). Es más, la mayoría de los relojes (¿me atrevería a decir todos?) no van en una dirección: normalmente incorporan (en un mínimo irreductible) algún elemento recíproco o resonante, y las características unidireccionales son un artificio auxiliar. Tampoco es el caso de que el tiempo parezca ser constante: todos los relojes conocidos son capaces de variar en condiciones ambientales.
@Steve. Creo que estás complicando la parte equivocada. El tiempo (con T mayúscula aquí) parece existir. Casi toda la física (y muchas otras cosas) tienen una parte de sus ecuaciones que tiene que ver con el Tiempo. Si medimos el Tiempo con muchos métodos diferentes, utilizando fenómenos subyacentes muy diferentes y todos concuerdan en los errores de medición, me parece que estamos muy cerca de observar y medir el Tiempo. Hasta nuevo aviso, diría que a todos los efectos lo que mide el Reloj puede decirse que es Tiempo. (Un análogo, creemos que la masa existe y la medimos).
@ghellquist, como digo, no se mide el puro paso del tiempo. No estoy argumentando que el tiempo sea eliminado de las ecuaciones. He argumentado acerca de qué hora es en realidad. Como experimento mental, vea si puede concebir un reloj que no tenga partes móviles, es decir, algún elemento crucial que se mueva en el espacio.
@Steve Descanso mi caso. "Si camina como un muelle y grazna como un muelle, probablemente sea un muelle". ¡Buena suerte con tu experimento!
@Steve: ¿está implícito en su pregunta que hay alguna verdad objetiva sobre cuál de un par de relojes funciona más lentamente en relación con el "tiempo"? Si es así, entonces la simple razón por la que los físicos no piensan de esta manera es porque parece haber una simetría completa en la forma en que funcionan las leyes de la física cuando se expresan en las coordenadas de diferentes marcos inerciales, y diferentes marcos inerciales pueden estar en desacuerdo sobre cuál de ellos. dos relojes corren más lento (y también discrepan acerca de la simultaneidad, es decir, si un par de eventos ocurrieron en el 'mismo tiempo' o en 'tiempos diferentes').
@Hypnosifl, mi pregunta (ahora fechada) no se trataba de un par de relojes; no es una paradoja de gemelos disfrazada y, por lo tanto, su comentario es irrelevante. Pero diré esto de todos modos en respuesta. No es físico y, francamente, el colmo del absurdo afirmar que ambos relojes funcionan más lento que el otro . Si ambos parecen ser más lentos que el otro, entonces se debe a retrasos en la señalización: la relación única entre los relojes debe ser de igualdad o de desigualdad en direcciones opuestas (uno más rápido y otro más lento). Por supuesto, cualquier observación dada puede consistir en ambos efectos.
No estaba sugiriendo que su pregunta fuera específicamente sobre un par de relojes, solo estaba respondiendo a la pregunta general de si está implícito en su pregunta que hay alguna verdad objetiva sobre el grado en que un reloj dado se está atrasando. En la relatividad no hay una verdad objetiva, por lo que no hay paradoja en la que cada uno "corra más lento que el otro" en ningún sentido objetivo, cada uno es más lento que el otro en el marco del otro, como si tuviera dos líneas no paralelas y le diera a cada una una coordenada espacial. sistema donde su propia pendiente es cero, en el sistema de cada uno el otro tiene una pendiente mayor.
@Hypnosifl, si no hay una verdad objetiva, entonces ya no estás hablando de ciencia. La única explicación objetiva posible de lo que describe son los efectos de señalización no modelados. Digo sin modelar: de hecho, están completamente modelados por la relatividad, pero hay un error de atribución en su interpretación. El error está en tratar "la pendiente" como si representara algo puramente sobre el reloj que se observa, en lugar de una conjunción de algo sobre el reloj que se observa y también algo sobre la señal del reloj que se observa (una combinación de dos variables físicas ) .
Hay una verdad objetiva sobre muchas cantidades en relatividad, "la tasa de tictac de un reloj" simplemente no es una de ellas. Tomando mi ejemplo con dos líneas en un plano 2D (que no tenía nada que ver con los relojes excepto por analogía), ¿está de acuerdo en que dos sistemas de coordenadas cartesianas xy diferentes pueden estar en desacuerdo sobre qué línea tiene la mayor pendiente y que no hay una verdad objetiva? sobre la pendiente independiente de la elección de los ejes de coordenadas? En cuanto a su comentario sobre los relojes, el cálculo de la velocidad de un reloj de cada cuadro se basa en factorizar los retrasos de la señal, asumiendo que la luz se mueve en c en ese cuadro.
(cont.) la velocidad a la que parece estar funcionando un reloj depende de la ecuación doppler relativista , y es diferente de la velocidad a la que se calcula que está funcionando en su cuadro una vez que se tiene en cuenta el hecho de que las señales de luz de diferentes tictacs de un reloj se mueven En relación a ti, tuviste que viajar diferentes distancias en tu marco para llegar a tus ojos. La tasa que obtiene cuando elimina los retrasos de la señal es la dada por la ecuación de dilatación del tiempo.
@Hypnosifl, dos sistemas de coordenadas diferentes pueden describir una pendiente diferente, sí, pero es posible que no se prefieran por igual. El problema tampoco es un mero retraso de la señal basado en la distancia; de hecho, en un tramo de entrada, en realidad es un avance de la señal que debe corregirse, lo que no es atribuible a la distancia, sino a la velocidad. Esto es observable como Doppler. Un tratamiento geométrico bidimensional estándar (diagrama de espacio-tiempo) de las trayectorias no "elimina" la velocidad; la relatividad se emplea específicamente para modelar sus efectos.
También estoy de acuerdo contigo en que una vez que se eliminan todos los efectos de distancia y velocidad, te queda la dilatación del tiempo. Pero en la paradoja estándar de los gemelos (un viaje de ida y vuelta a la Tierra), esa dilatación solo la sostiene un reloj, y ambos están de acuerdo en cuál (es decir, no es absurdo que de alguna manera ambos relojes estén uno detrás del otro).
En la paradoja de los gemelos, existe la asimetría de que uno tiene que acelerar para dar la vuelta y así no permanece en reposo en ningún marco inercial, mientras que el otro sí. Pero puede calcular el tiempo total transcurrido en ambos relojes desde la perspectiva de cualquier marco inercial que desee, incluidos los que no están de acuerdo sobre qué reloj avanzaba más lento en cualquier tramo del viaje, aunque todos los marcos obviamente están de acuerdo en que el gemelo que dio la vuelta tenía una tasa promedio más lenta .
Y la corrección Doppler se puede derivar únicamente de los retrasos de la señal en función de la distancia. Puedo dar un ejemplo numérico si lo desea. En cuanto al ejemplo con diferentes pendientes, ¿por qué cualquiera de los dos sistemas de coordenadas cartesianas sería "preferido"?
@Hypnosifl, no hay "desacuerdo" sobre cuáles son más lentos en cualquier pierna. O vuelve a describir los efectos atribuibles a la señalización (no al estado de los relojes en sí) o está empleando una comparación con un marco inválido (y usted mismo dice que el despegue y la inversión de un gemelo muestra que cualquier el marco que permanece ubicado junto a él en todo momento es un marco inválido en el que basar cualquier comparación u obtener resultados). Una corrección Doppler debe basarse en la velocidad, no solo en la distancia (aunque me pregunto si su declaración al respecto fue confusa).
La corrección Doppler es, por supuesto, una función de la velocidad, pero mi punto es que se puede derivar simplemente asumiendo que en mi marco, el intervalo de tiempo "real" entre los tictacs de un reloj en movimiento está dado por la ecuación de dilatación del tiempo, y luego considerando las diferentes distancias que el reloj estaría de mí en cada tic debido a su movimiento en mi marco, luego, al agregar los diferentes retrasos de la señal, se obtiene el intervalo de tiempo visual entre tics, que es diferente del "real". Entonces, en ese sentido, está derivando Doppler solo de la dilatación del tiempo + el retraso de la señal.
(cont.) Y hay un desacuerdo sobre cuál corre más lento en cada pierna, debido a la relatividad de la simultaneidad: si la primera pierna comienza cuando un gemelo deja la tierra y termina cuando acelera para dar la vuelta y regresar a la tierra, ambos marcos están de acuerdo en cuánto tiempo ha pasado en su reloj entre esos eventos, pero no están de acuerdo en qué tic del reloj en el reloj del gemelo de la Tierra es simultáneo con el evento del cambio, por lo que no están de acuerdo en cuánto ha envejecido el gemelo de la Tierra "al mismo tiempo". tiempo" que el gemelo viajero da la vuelta.
(cont.) Puede usar un marco en el que el gemelo viajero haya envejecido más que el gemelo terrestre en el momento del cambio, no menos, pero este marco dirá que en el segundo tramo donde el gemelo viajero está de regreso a la Tierra, envejece mucho más lentamente que el gemelo de la Tierra en ese segundo tramo, por lo que este marco obtiene exactamente la misma predicción que cualquier otro marco acerca de que el gemelo que viaja es más joven en el momento en que regresa a la Tierra (comparación local de relojes es objetivo e independiente del marco, pero las tasas instantáneas de tictac para relojes distantes no lo son)
@Hypnosifl, no hay ninguna razón por la cual el Doppler no deba derivarse de la dilatación del tiempo, porque los dos están en una relación fija (cuando se usa un marco apropiado). No hay ningún desacuerdo genuino, porque en la configuración estándar solo un gemelo tiene un marco de movimiento conjunto legítimo en todo momento: el otro es un marco no válido y sus afirmaciones son falsas (y por lo tanto, no hay importancia en su "desacuerdo" con el análisis correcto, porque deberíamos esperar que las respuestas incorrectas no estén de acuerdo con las respuestas correctas).
Además, persiste en afirmar que cada uno de ellos "no está de acuerdo" en cuanto a qué reloj avanza más lento en cada etapa. De hecho, no lo hacen. En un análisis correcto, ambos descubren que el gemelo viajero es aquel cuyo reloj está atrasado. La observación ingenua que ambos hacen de que el otro está perdiendo tiempo en el tramo de ida (y ganándolo en el de vuelta), es el elemento Doppler de la observación, no el elemento de dilatación del tiempo (que sólo acumula el gemelo que viaja).
"el otro es un marco inválido, y sus afirmaciones son falsas" Si está hablando de un marco en el que el gemelo que viaja está en reposo, entonces sí, ese no es un marco inercial, por lo que no puede usar las reglas de marcos inerciales . Pero eso no es de lo que estaba hablando, estaba señalando que eres libre de analizar las cosas desde la perspectiva de un marco inercial en el que el gemelo que viaja tiene una velocidad menor que el gemelo de la Tierra durante el primer tramo, pero un mayor velocidad durante el segundo tramo. En tal marco, el gemelo que viaja envejece más rápido durante el primer tramo, más lento en el segundo tramo.
Vea, por ejemplo, el diagrama aquí , que muestra los mismos caminos del espacio-tiempo desde la perspectiva de dos marcos de inercia diferentes, uno donde el gemelo de la Tierra permanece en reposo todo el tiempo, otro donde el gemelo que viaja está en reposo durante el primer tramo pero no el segundo. Podría pensar en esto como el marco de reposo de un tercer observador inercial que inicialmente está en reposo en relación con el gemelo que viaja en el primer tramo, pero que no cambia de velocidad cuando lo hace el gemelo que viaja. En este diagrama las líneas horizontales son líneas de simultaneidad en cada cuadro.
@Hypnosifl, puede usar (lo que podemos llamar) un "tercer cuadro", pero (como reconoce) todavía no hay desacuerdo sobre quién termina siendo el más joven y de quién es el reloj. Las piernas que está observando desde este tercer marco no son el estado del reloj viajero, está observando las señales recibidas del reloj viajero y, naturalmente, esas señales están en una configuración diferente nuevamente que en el marco de la Tierra. Su problema es que no está haciendo distinción entre el estado del reloj en sí y el estado de la señal que recibe de él (y cuándo).
No puedo enfatizar lo suficiente que el hecho mismo de este cambio supuestamente extraño de las tasas de envejecimiento, cambiando en ambas direcciones, con el envejecimiento aparentemente lento y acelerado en diferentes piernas, ¡es un caso clásico de un efecto de señalización! Sus edades realmente no están cambiando de velocidad (excepto por la pequeña desaceleración sostenida experimentada por el gemelo que viaja). Puede demostrar los mismos efectos relativistas con un sistema de señalización sonora.
"Las piernas que está observando desde este tercer cuadro no son el estado del reloj viajero, está observando las señales recibidas del reloj viajero" No estoy seguro de lo que quiere decir con esto: todavía hay una distinción entre el ritmo de los relojes de el gemelo viajero y el gemelo de la Tierra están "realmente" corriendo en este tercer cuadro y la velocidad a la que visualmente parecerían estar corriendo para un observador en reposo en este cuadro, con el primero dado por la ecuación de dilatación del tiempo y el último por la ecuación de Doppler relativista. ¿Estás de acuerdo con eso?
También tenga en cuenta que en las explicaciones de la relatividad es tradicional definir las coordenadas de tiempo en las que suceden los eventos en cada cuadro en términos de mediciones locales por una cuadrícula de reglas y relojes como se muestra en el diagrama aquí , con los relojes sincronizados por la convención de sincronización de Einstein . . Por lo tanto, no habría necesidad de preocuparse por los retrasos en la señal, el observador puede simplemente observar la hora en cualquier reloj junto al que esté pasando uno de los gemelos.
(continuación) Digamos que ambos gemelos tenían sus relojes puestos a cero en el momento en que partieron, y el reloj en la cuadrícula del tercer observador que estaba junto a ellos en este momento también mostraba una lectura de 0. En el escenario del diagrama, el tercer observador observaría que cuando el reloj del gemelo viajero marcaba 4 años, él estaba junto a un reloj en la cuadrícula que también marcaba 4 años; sin embargo, cuando el reloj del gemelo de la Tierra marcaba 3,2 años, ese fue el momento en que pasó junto a un reloj en la cuadrícula del tercer observador que marcaba 4 años. Esa es la base para decir que en este marco, el gemelo de la Tierra envejeció más lentamente en el partido de ida.
@Hypnosifl, el análisis (del tipo ilustrado por un diagrama de espacio-tiempo) realizado desde el tercer cuadro no es la velocidad a la que los relojes están "realmente" funcionando. Es la velocidad a la que parecen estar marcando (para el equipo receptor que se mueve con el marco). Esta apariencia consiste tanto en la dilatación real (del tipo sostenido solo por un gemelo) como en la variación mediada por señalización (que es tanto dilatatoria como contractiva en diferentes piernas, piernas que también parecen comenzar y terminar en diferentes momentos para diferentes observadores debido a efectos de señalización).
La razón por la que las piernas comienzan y terminan en diferentes momentos para diferentes marcos es obviamente porque el gemelo que realmente cambia de rumbo cambia inmediatamente su relación con otras señales: para él, la señal recibida de la Tierra cambia inmediatamente (hablando en forma abreviada...) de rojo a azul. Pero para el gemelo de la Tierra que está estacionario, no se entera inmediatamente de la inversión de curso en el momento en que ocurre: las señales del gemelo que viaja solo cambian de rojo a azul más tarde, algún tiempo después de que realmente sucedió, y por momento en el que el gemelo viajero ya está en camino de regreso.
Hablas de sincronización con el tercer cuadro, asumiendo que hay un "momento" cuando ocurre la sincronización. De hecho, no existe tal momento, porque la sincronización (a través de un medio de onda con frecuencia/longitud de onda finita) requiere un intervalo. Nunca es solo una cuestión de la distancia instantánea entre las partes móviles: se debe modelar la dinámica de cuánto se mueven durante la sincronización, porque cualquier señal que se envíe se verá modificada por el movimiento. Eso es lo que hace la relatividad.
Dije '"realmente" tictac en este tercer marco' (no en ningún sentido objetivo), en oposición a la tasa de tictac vista visualmente por el observador en ese marco, que depende del efecto Doppler. No respondiste mi anterior "¿estás de acuerdo con eso?" pregunta sobre si entiende que estos son distintos, y si por "parece estar haciendo tictac (para recibir el equipo que se mueve con el marco)" solo se refiere a una cámara sostenida por un observador en reposo en ese marco, ese sería el valor desplazado por Doppler , no el valor "real" en ese marco definido por observaciones locales en relojes sincronizados en ese marco.
Como digo, no está haciendo una distinción entre la imagen que describe la relatividad para un receptor determinado y la realidad del estado del sistema. Las edades de los gemelos no se balancean por todas partes durante el viaje. Puede parecer que lo hacen debido a los efectos relativistas sobre las señales mediante los cuales observamos sus edades (y esos efectos son relativos a cualquier par dado de emisores y receptores). La relatividad modela esos efectos. Aparentemente, su error es pensar que esos modelos están describiendo la realidad cambiante de las edades de los gemelos.
asumiendo que hay un "momento" cuando ocurre la sincronización No tengo idea de lo que quiere decir con esto, así que asumo que está malinterpretando algo. La sincronización de Einstein es solo un procedimiento físico para establecer las horas en los relojes en el tipo de cuadrícula que describí, que implica activar destellos de luz entre pares de relojes y asegurarse de que muestren la misma hora en el momento en que la luz los alcanza. Supongamos que esto se hizo mucho antes del experimento de los gemelos, cuando no importa, lo único que importa es que los relojes de la cuadrícula permanezcan sincronizados entre sí en este marco durante todo el experimento.
no está haciendo una distinción entre la imagen que describe la relatividad para un receptor determinado y la realidad del estado del sistema. Mi pregunta no se trata de sus creencias sobre la "realidad del estado del sistema", solo estoy preguntando si entiende que en la teoría de la relatividad en sí misma (incluso si cree que es incorrecta), hay una diferencia entre A) qué tan rápido parecen estar corriendo los relojes visualmente para un observador en el tercer cuadro que recibe señales, y B) qué tan rápido rápido están marcando las coordenadas de ese marco que se puede definir mediante mediciones locales en una cuadrícula de reloj.
Las edades de los gemelos no se balancean por todas partes durante el viaje. Tampoco estoy seguro de lo que quiere decir con esto, el tercer cuadro no dice que la edad de nadie cambie de forma salvaje, dice que durante los primeros cuatro años de tiempo coordinado, el viaje el reloj del gemelo transcurre constantemente 4 años mientras que el reloj del gemelo terrestre transcurre constantemente 3,2 años, y luego durante los siguientes 8,5 años de tiempo coordinado, el reloj del gemelo viajero transcurre constantemente 4 años más mientras que el reloj del gemelo terrestre transcurre constantemente 6,8 años más, así que cuando el reúnen el viajero ha transcurrido 4+4=8 y la Tierra gemela 3,2+6,8=10.
@Hypnosifl, el proceso que describe (de simplemente poner a cero la progresión de los relojes cuando cada uno recibe un pulso) no los sincronizará si, para empezar, los relojes no avanzan a la misma velocidad. Y sabemos que los relojes no son estables bajo el movimiento de inercia, la evidencia experimental lo demuestra. Es por eso que el gemelo regresa más joven: su reloj realmente funciona más lento que el de la Tierra. No es la relatividad lo que está mal, es tu comprensión de lo que está describiendo. Estoy de acuerdo con su afirmación A/B (si la he entendido correctamente).
La pregunta es cuáles son las edades reales de los dos gemelos en un momento dado. Usted es incapaz de describir una respuesta que no vuelva a describir disparidades amplias y variables; en otras palabras, una descripción que reconozca que 3,2 años en el tramo de ida y 6,8 en el de regreso no es la progresión variable en la edad del gemelo de la Tierra, sino la progresión variable en la velocidad a la que se reciben las señales sobre la edad del gemelo de la Tierra. Tienes que meterte en la cabeza que esta distinción que no haces es fundamental para la mayoría de la gente.
"no los sincronizará si, para empezar, los relojes no avanzan a la misma velocidad". La sincronización de Einstein se trata solo de sincronizar los relojes en una cuadrícula rígida del tipo que se muestra en la ilustración entre sí : todos están en reposo entre sí, lo que significa que todos los marcos de inercia los definirán para tener la misma velocidad entre sí. (y ninguno de ellos cambia la velocidad), por lo que todos los marcos de inercia estarán de acuerdo en que están marcando "a la misma velocidad".
Es decir, hay una diferencia entre lo que se observa que es el estado en un momento dado y lo que realmente es en un momento dado; la diferencia se explica potencialmente por las limitaciones de la señalización y la transferencia de información. Nadie está obligado a suscribir su filosofía, y la gran mayoría de los científicos e ingenieros (a excepción de una minoría de físicos) la rechazan. No es una filosofía sobre la que descansa el formalismo de la relatividad, sino una elección de rechazar una distinción que otros insisten en que es importante y pertinente para la forma en que se describe el mundo.
(cont.) Otros marcos inerciales dirán que este procedimiento da como resultado que los relojes en la cuadrícula no estén sincronizados, no porque sus tasas de tictac se vean como diferentes, sino simplemente porque los relojes se están moviendo en estos otros marcos, entonces si activa un destello de luz entre un par, un reloj se moverá hacia la señal y el otro alejándose de ella, por lo tanto, en el otro marco, la luz debe incidir en los dos relojes en momentos diferentes.
@Hypnksifl, el "tercer cuadro" que describió no estaba "en reposo" con respecto a la Tierra, y entendí que se refería a la sincronización con un reloj que se mueve con ese cuadro. Creo que nos podemos estar confundiendo debido al gran volumen de texto y ahora a la mezcla de respuestas. El tema controvertido no es sincronizar relojes que están en reposo entre sí, sino describir un sistema con partes móviles y relojes que no están en reposo.
"La pregunta es cuáles son las edades reales de los dos gemelos en un momento dado". Pero la relatividad niega la existencia de cualquier "momento" objetivo, ya que eso presupondría algún tipo de verdad objetiva sobre qué pares de eventos ocurren simultáneamente. Incluso si crees que existe tal verdad, ¿cómo la determinarías? ¿Afirma usted que en un escenario de paradoja de gemelos, la definición de simultaneidad del gemelo inercial es siempre la correcta, incluso si el gemelo inercial se mueve a una alta velocidad en relación con la Tierra y el gemelo con la trayectoria no inercial está inicialmente en reposo en el ¿Tierra para el partido de ida?
"El tema controvertido no es sincronizar relojes que están en reposo entre sí, sino describir un sistema con partes móviles y relojes que no están en reposo". Mi punto es que puedes describir eventos en otros sistemas (como los gemelos) asignando coordenadas de tiempo a los eventos utilizando medidas locales en su cuadrícula de reloj, por lo que no hay que preocuparse por los retrasos en la señal. por ej. el tercer observador puede decir "cuando el reloj del gemelo no inercial marcaba 4 años, estaban junto a un reloj cuadriculado que marcaba 4 años" y "cuando el reloj del gemelo terrestre marcaba 3,2 años, estaban junto a un reloj cuadriculado que marcaba 4 años".
Y sí, estoy de acuerdo en que, dada una cuadrícula de relojes en 3D, cualquier movimiento de un receptor dará como resultado que la cuadrícula de relojes parezca no estar sincronizada entre sí. Esto se debe a que la distancia entre el receptor en movimiento y todos los relojes de la cuadrícula no pueden tener una dinámica simétrica. Es decir, las derivadas de la distancia entre el receptor y cada reloj es diferente para cada reloj (o al menos, para ser quisquilloso, y dependiendo de la configuración espacial, hay al menos dos conjuntos de relojes, y la dinámica de cualquiera de los dos relojes no en el mismo conjunto, no son iguales entre sí).
"Eso se debe a que la distancia entre el receptor en movimiento y todos los relojes de la cuadrícula no pueden tener una dinámica simétrica". Sí, una cuadrícula 3D de relojes sincronizados usando el procedimiento de Einstein aparecerá fuera de sincronización en el segundo cuadro donde los relojes se mueven, pero no tiene que pensar en eso en términos de un "receptor" en una ubicación específica, puede pensar en términos de que el segundo cuadro tiene su propia cuadrícula de reloj que usan para asignar coordenadas a eventos usando observaciones puramente locales; vea los diagramas que publiqué aquí .
"Pero la relatividad niega la existencia de cualquier "momento" objetivo" - no, su interpretación realiza esa negación. La existencia de un momento universal es un axioma en la ciencia y la ingeniería, y la relatividad está totalmente en consonancia con ella. Lo que es relativamente simultáneo en relatividad es la recepción de eventos de señalización. No afirmo que el marco del gemelo inercial sea siempre el correcto; de hecho, afirmo que el marco del universo inercial es el correcto (es decir, afirmo la existencia de un marco preferido). Eso corresponde aproximadamente al marco de inercia de la Tierra.
"Por ejemplo, el tercer observador puede decir "cuando el reloj del gemelo no inercial marcaba 4 años, estaban al lado de un reloj de cuadrícula que marcaba 4 años". El problema es que en los sistemas donde las partes móviles se mueven a velocidades cercanas a el límite de velocidad del método de señalización, es decir, donde los relojes pasan uno junto al otro a velocidades relativistas, se vuelve cada vez más difícil definir "al lado", porque cualquier señal del reloj en movimiento podría extenderse durante eones. Puede corregir esto con la relatividad, pero eso plantea la misma pregunta sobre la que discutimos, que es lo que describe la relatividad.
"tu interpretación realiza esa negación" Para ser claros, no estoy defendiendo ninguna interpretación filosófica, cuando digo que no hay un presente objetivo quiero decir que si diferentes personas definen la simultaneidad de manera diferente, no hay una prueba empírica que pueda establecer quién tiene razón . Eres libre de tomar una interpretación filosófica que acepte esto pero diga que hay una verdad metafísica sobre qué eventos son realmente simultáneos, esto sería una verdad completamente invisible para todos los experimentos posibles, como las variables ocultas en la interpretación de Bohm de la mecánica cuántica.
"La existencia de un momento universal es axiomática en ciencia e ingeniería" ¿Cómo es eso? Si, en cambio, imagina los eventos como si sucedieran en un espacio-tiempo 4D que se puede describir en una variedad de sistemas de coordenadas sin que se realice el eje de tiempo de un sistema (así como estoy seguro de que reconocería que los científicos son libres de usar diferentes ejes xyz para describir eventos en el espacio 3D y que no existe una forma 'objetivamente correcta' de orientar estos ejes), ¿cómo afectará esto su capacidad para realizar experimentos científicos en la práctica?
"De hecho, afirmo que el marco del universo inercial es el correcto (es decir, afirmo la existencia de un marco preferido)". No sería "preferido" en el sentido en que los físicos usan el término, es decir, las leyes de la física no lo harían. funcionan de manera diferente cuando se describen en las coordenadas de este marco que en cualquier otro. Además, supongo que esto significa que en el caso de que el gemelo no inercial permanezca en reposo en la Tierra durante el primer tramo, en realidad estaría de acuerdo con los números que di donde el gemelo inercial solo tiene 3.2 años envejecidos en el momento en que el gemelo no inercial -gemelo inercial acelera después de experimentar 4 años?
"Se vuelve cada vez más difícil definir 'junto a', porque cualquier señal del reloj en movimiento podría extenderse durante eones" No estoy seguro de lo que quiere decir con esto: ¿de qué "señal" está hablando? Si hay una cámara en un reloj grabando continuamente imágenes del segundo reloj cuando hace un pase cercano (dentro de unos pocos centímetros, digamos), el pase pasaría muy rápido, y cualquiera podría mirar más tarde las imágenes en su tiempo libre para ver qué la lectura en el segundo reloj fue en el momento de su mayor acercamiento a la cámara, no hay señales de larga distancia que no sean la transmisión de imágenes locales.
En realidad, no eres libre de usar diferentes coordenadas 3D. Una pregunta obvia es si la Tierra o el Sol es el centro del sistema solar. Su filosofía dice que la respuesta es una elección según cada uno, careciendo de la coherencia sistemática incluso de quien afirma firmemente el geocentrismo. Mi punto no es condenar la vista 4D, en realidad es una herramienta enormemente útil, sino ser claro sobre lo que describe.
Usted pregunta *"¿cómo estropeará su capacidad para realizar experimentos científicos en la práctica?", Sospecho que la terrible verdad del asunto es que una visión incorrecta le impedirá incluso concebir experimentos pertinentes, y creará una bola de confusión que frustra tanto la reproducción como el avance de la ciencia. Sabíamos que una visión esencialmente lorentziana era defendible todo el tiempo y totalmente acorde con la relatividad (el mismo Einstein lo dijo). Sabemos desde los años 60 sobre el CMB, que fue después de la muerte de la gran mayoría de los físicos importantes como Einstein.
"En realidad, no eres libre de usar diferentes coordenadas 3D". Solo estaba hablando de la orientación de los ejes x, y, z (a los que me referí en mi comentario), no del movimiento del origen a lo largo del tiempo. ¿Crees que hay algún tipo de dirección "objetiva" para el eje x, de modo que hay una verdad objetiva acerca de si dos eventos comparten la misma coordenada x, y cualquiera que oriente su eje x de manera diferente está equivocado? Si no, no veo cómo se interpondría en el camino de hacer buena ciencia negar también que haya un eje t objetivo o una verdad objetiva acerca de si dos eventos comparten la misma t-coord.
"Sabemos desde los años 60 sobre el CMB, que fue después de la muerte de la gran mayoría de físicos importantes como Einstein" Sí, pero creo que encontrará que los científicos que lo descubrieron y continuaron investigando no lo hicieron. creo que demostró que la relatividad de la simultaneidad es incorrecta, lo que sugiere que asumir la relatividad de la simultaneidad en realidad no se interpone en el camino de hacer buena ciencia en un nivel práctico.
"las leyes de la física no funcionarían de manera diferente cuando se describen en las coordenadas de este marco que en cualquier otro" - la razón por la cual las leyes locales de la física son las mismas en cualquier marco inercial es porque el reloj local en sí está sujeto a la variación inducida por cualquier movimiento. De eso se trataba mi pregunta (arriba), preguntando por qué una minoría de científicos en física se ha preocupado por la creencia ideológica de que las velocidades de los relojes atómicos son constantes, cuando la evidencia experimental dice precisamente lo contrario.
"Si hay una cámara en un reloj grabando continuamente imágenes del segundo reloj mientras hace un pase cercano (dentro de unos pocos centímetros, digamos), el pase pasaría muy rápido" - ¡no ! Por el contrario, a medida que aumentan las velocidades, una longitud de onda (el cuanto mínimo de señalización) se vuelve cada vez más larga y tarda cada vez más en recibirse. No solo se desplaza hacia el rojo, la ubicación real de su emisión se borra a lo largo del camino (porque el reloj en movimiento está pasando a toda velocidad, no está en el mismo lugar al final de un ciclo que al principio).
"¿Crees que hay algún tipo de dirección 'objetiva' para el eje x?" - no, pero tampoco puedo ver nada que sea pertinente a la cuestión de cómo se orientan los 3 ejes espaciales. El problema en su análisis no es solo que "los eventos no comparten la misma coordenada t", sino que hace afirmaciones adicionales específicas sobre lo que describen estas coordenadas, lo que luego conduce a una contradicción (como "el reloj de cada uno es más lento que el otro", donde se entiende que se trata de una afirmación sobre la relación entre dos estados locales, no sobre la relación entre un estado local y una señal local recibida).
"Sí, pero creo que encontrará que los científicos que lo descubrieron y continuaron investigando no creían que probara que la relatividad de la simultaneidad es incorrecta". Yo tampoco lo he declarado incorrecto, es como Como mencioné antes, el punto crucial es qué se considera exactamente que son los "eventos" en este esquema. ¿Qué es lo que se considera relativamente simultáneo? ¿El acontecimiento mismo o la recepción de una señal que informa a un receptor sobre el acontecimiento?
"¡no! Por el contrario, a medida que aumentan las velocidades, una longitud de onda (el cuanto mínimo de señalización) se vuelve cada vez más larga" Solo si la fuente se aleja de ti, se vuelve cada vez más corta si se acerca a ti, así que en mi experimento mental un reloj tiene una cámara, esto facilitaría la identificación del punto de mayor aproximación ya que el reloj en movimiento cambiaría visualmente de azul y tictac rápido a rojo y tictac lento. Esto también significa que en la fase de aproximación hasta la aproximación más cercana, los fotones podrían detectarse en escalas de tiempo más cortas que si el reloj pasara lentamente.
(en cualquier caso, el diagrama de paradoja gemela original al que me vinculé solo involucra velocidades relativas de hasta 0.88 la velocidad de la luz en relación con el tercer observador, el factor de dilatación del tiempo nunca supera los 2.125 aquí, por lo que no hará mucha diferencia para el minúsculo tiempo necesario para detectar localmente fotones individuales en la longitud de onda visible)
'hace afirmaciones adicionales específicas sobre lo que describen estas coordenadas, lo que luego conduce a una contradicción (como "el reloj de cada uno es más lento que el otro', ya señalé que no hago tal afirmación en un sentido objetivo independiente del marco, solo que cada reloj es más lento en las coordenadas del marco de reposo del otro reloj. Esto no es más una contradicción que las dos líneas en el espacio 2D, cada una de las cuales usa un sistema de coordenadas xy donde el eje x es paralelo a sí mismo, de modo que cada línea tiene un mayor pendiente en el propio sistema xy de la otra recta.
'la razón por la que las leyes locales de la física son las mismas en cualquier marco inercial es porque el reloj local en sí mismo está sujeto a variaciones inducidas por cualquier movimiento': esa es la interpretación de la relatividad de Lorentz, que está bien siempre que reconozca que es metafísica reclamo y no hay una forma comprobable de determinar qué reloj "realmente" se está moviendo más rápido/marcando más lento. Su idea de que el marco "verdadero" es el CMB parece ser una cuestión de fe, podría haber un mundo posible donde había un marco verdadero pero no era el marco CMB, sin diferencias empíricas de uno donde estaba .
¿Qué es lo que se considera relativamente simultáneo? ¿El acontecimiento mismo, o la recepción de una señal que informa a un receptor sobre el acontecimiento? El acontecimiento mismo. Y de nuevo, la hora del suceso siempre se puede definir en términos de algo así como imágenes de cámara del suceso de un reloj de cuadrícula que estaba arbitrariamente cerca cuando ocurrió. Sin embargo, como siempre, no se afirma si un par de eventos son simultáneos en algún sentido metafísico "verdadero", solo si se les asigna el mismo tiempo en un marco de referencia definido físicamente.
@Hypnosifl, "se vuelve cada vez más corto si se mueve hacia usted" , de hecho, pero luego tiene el problema opuesto, que la señal del reloj estacionario ahora es irracionalmente larga en comparación, y el reloj en movimiento marca muchas veces en muchos lugares para un tictac del reloj estacionario. En realidad, es una situación simétrica, el problema fundamental es que el movimiento relativo hace que el intervalo de señalización de un reloj sea inconmensurable con el otro (cuál de ellos se alarga o se acorta no es el problema, es la desigualdad de ellos).
"pero entonces tienes el problema opuesto, que la señal del reloj estacionario ahora es irrazonablemente larga en comparación" ¿Desde la perspectiva de quién? No desde la perspectiva del propio reloj estacionario obviamente, pero tampoco desde el 2° reloj mientras está en fase de aproximación hasta el momento de la pasada más cercana, ya que desde la perspectiva del 2° reloj el 1° reloj es el que se acerca rápidamente en esa fase.
"Esa es la interpretación de Lorentz de la relatividad, que está bien siempre y cuando se reconozca que es una afirmación metafísica y que no hay una forma comprobable de determinar qué reloj "realmente" se mueve más rápido o más lento". - No tengo ningún problema en reconocer eso, el problema es que el tuyo es igualmente metafísico cada vez que intenta enunciar una interpretación física significativa del formalismo.
"el tema es que el tuyo es igualmente metafísico" ¿Puedes señalar un comentario mío que interpretas como una afirmación metafísica? Creo que no entendiste mi significado, mira mi comentario anterior donde dije "Para ser claro, no estoy defendiendo ninguna interpretación filosófica, cuando digo que no hay un presente objetivo quiero decir que si diferentes personas definen la simultaneidad de manera diferente, no hay prueba empírica que pueda determinar quién tiene la razón". Y antes de entrar en esta discusión con usted, hice el mismo comentario sobre la física frente a la metafísica en un comentario en el hilo del foro de filosofía al que se vinculó aquí.
"Su idea de que el marco "verdadero" es el CMB parece ser una cuestión de fe", no más de lo que es una cuestión de fe decir que el sol es el centro del sistema solar. Hay una realidad física objetiva para el CMB que no es una cuestión de elección de marcos. Si se debe elegir un solo marco para describir el mundo de una manera mutuamente inteligible, entonces parece un candidato razonable.
"¿Desde la perspectiva de quién?" - desde la perspectiva de la cámara que se agregó al escenario.
"¿Puedes señalar un comentario mío que interpretas como una afirmación metafísica?" - aquel en el que afirma que la relatividad describe el estado real del otro reloj (es decir, cuando dice parafraseado "cada reloj está atrasado según el otro, y esto no es un retraso de señalización"). O cuando dices "no hay presente objetivo". Estas son afirmaciones filosóficas de rango.
Un reloj está en reposo en relación con la cámara, por lo que sus señales de luz son las pequeñas longitudes de onda normales que encontramos en la vida visible en la vida cotidiana, el otro se mueve hacia la cámara en la fase de aproximación, por lo que sus señales de luz tienen una longitud de onda aún más corta. ¿En qué sentido son las señales de cualquiera de las cámaras "irrazonablemente largas" desde la perspectiva de la cámara, lo suficiente como para dificultar la determinación local de la hora del reloj de cada uno en el acercamiento más cercano? (hasta el microsegundo más cercano, digamos; obviamente, si exige una resolución de tiempo lo suficientemente pequeña, incluso las longitudes de onda normales de luz visible pueden ser demasiado largas)
Con ambas afirmaciones, inmediatamente aclaré que no tenía un significado filosófico en mente, ¿te lo perdiste? Con el primero dije "Ya señalé que no hago tal afirmación en un sentido objetivo independiente del marco, solo que cada reloj es más lento en las coordenadas del marco de reposo del otro reloj". Con el segundo dije "Para ser claro, no estoy defendiendo ninguna interpretación filosófica, cuando digo que no hay un presente objetivo quiero decir que si diferentes personas definen la simultaneidad de manera diferente, no hay una prueba empírica que pueda determinar quién tiene razón".
"no más de lo que es una cuestión de fe decir que el sol es el centro del sistema solar" No creo que ningún físico diría "el sol es el centro del sistema solar" en ningún sentido metafísico (refiriéndose a espacio absoluto, digamos), si dijeran esa frase estarían pensando en alguna definición física comprobable específica, como centro de masa en una aproximación newtoniana. Y en la relatividad general no existe un único centro físicamente correcto, como lo señala aquí el astrónomo Phil Plait .
"¿En qué sentido las señales de cualquiera de las cámaras son 'irrazonablemente largas' desde la perspectiva de la cámara?", ¡ Porque el propósito de la cámara es establecer una sincronización de los malditos relojes! La dificultad aquí es que, a pesar del hecho de que los relojes tienen el mismo diseño y funcionan a las mismas velocidades, el efecto del movimiento relativo es hacerlos no sincronizables. Es decir, cualquier intento de lograr una sincronía instantánea seguirá colapsando en la insincronía. La insincronía puede ser en cualquier dirección. Cualquier señal recibida debe perder sincronía.
"Con ambas afirmaciones aclaré de inmediato que no tenía un significado filosófico en mente, ¿te lo perdiste?" - y no eres el primer acusado en afirmar "Yo no lo hice". El hecho de que niegue negociar con tales reclamos no significa que no lo esté haciendo. No se puede hacer ninguna afirmación sobre el estado de ningún reloj sin introducir una interpretación. ¡Todo lo que puede afirmar es que tiene números en papel, y eso solo es vacío!
"porque el propósito de la cámara es establecer una sincronización de los malditos relojes" No, no lo es, como dije, la sincronización fue solo entre relojes en la misma cuadrícula, que están todos en reposo entre . La cámara debe hacer una medición local de lo que dos relojes diferentes (uno en la cuadrícula y otro en movimiento relativo a la cuadrícula) cada uno lee en el evento específico (o "sucediendo" como usted dice) de su paso muy cerca de uno otro, esto no tiene nada que ver con "sincronizarlos" ya que el resultado de esta observación no se utilizará para reiniciar ninguno de los relojes.
"No se puede hacer ninguna afirmación sobre el estado de ningún reloj, sin introducir una interpretación" ¿Cómo se calcula? Si las predicciones numéricas sobre cosas como las lecturas locales de los relojes cuando pasan uno al lado del otro son acordadas por personas con diferentes interpretaciones filosóficas, obviamente no dependen de las interpretaciones. Si quiere llamar "vacío" a la predicción de resultados numéricos sin interpretación, básicamente está diciendo que toda la empresa científica es vacía a menos que agreguemos una metafísica que no tiene nada que ver con hacer predicciones falsables.
Creo que me estoy confundiendo con el escenario. ¿Es la idea en realidad que hay una cuadrícula de relojes a lo largo de todo el itinerario del viaje del gemelo viajero y, por lo tanto, dondequiera que esté, hay un reloj (estacionario en relación con la tierra y a una distancia fija) y una cámara capaz de registrar lo que lee el reloj del gemelo viajero en ese lugar?
"Si personas con diferentes interpretaciones filosóficas acuerdan predicciones numéricas sobre cosas como las lecturas locales de los relojes cuando pasan uno al lado del otro, obviamente no dependen de las interpretaciones", pero no estamos de acuerdo sobre cuáles son esas lecturas locales . son. "básicamente estás diciendo que toda la empresa científica es vacía a menos que agreguemos una metafísica que no tiene nada que ver con hacer predicciones falsables". - sí, creo que el falsacionismo es en sí mismo una tontería filosófica.
Sí, para definir el significado físico de las coordenadas de tiempo en el marco de la Tierra imaginamos una cuadrícula de relojes en reposo en el marco de la Tierra y cubriendo todo el volumen del espacio donde ocurre el experimento, por lo que el gemelo viajero está constantemente pasando cerca de diferentes relojes en el red. Para definir el significado físico del tiempo en algún otro marco (por ejemplo, el marco del tercer observador que está en reposo en relación con el gemelo que viaja durante el tramo de ida), también podemos imaginar una cuadrícula diferente de relojes en reposo en ese marco . La sincronización de Einstein solo ocurre entre relojes en la misma cuadrícula.
"pero no estamos de acuerdo en cuáles son esas lecturas locales". ¿Ni siquiera con mi aclaración sobre cómo se definen las lecturas en términos de observaciones locales en una cuadrícula de relojes que llenan el espacio? "sí, creo que el falsacionismo es en sí mismo una tontería filosófica". De acuerdo, simplemente sustituya "comprobable", hay otras nociones de cómo probamos teorías (métodos bayesianos, por ejemplo) que probablemente sean mejores que el falsacionismo; creo que todas tendrían la propiedad de que debería haber posibles resultados empíricos que cuenten. como evidencia contra una teoría (aunque no sea una 'falsificación' definitiva de la misma).
Así que tenemos la cuadrícula de relojes dispuesta como un túnel, y tenemos al gemelo viajero y su reloj volando a través de este túnel de relojes, y luego regresando. ¿Cuáles son los reclamos que se derivan de esta configuración?
Nuevamente, depende de qué cuadrícula de relojes elijamos, ya sea que elijamos uno en reposo con respecto a la Tierra o en reposo con respecto al tercer observador o algo más. Dado que objetó sobre lo que se observa en marcos distintos del marco de la Tierra, ¿puedo seguir adelante y asumir que estamos hablando de la cuadrícula que está en reposo en relación con el tercer observador (que está en reposo en relación con el viajero)? gemelo en el tramo de salida, pero continúa moviéndose inercialmente después de eso en lugar de acelerar junto con el gemelo que viaja)?
"Está bien, simplemente sustituya "comprobable"" : el problema es que la ciencia no se juzga puramente mediante experimentos, hay elementos filosóficos grandes e irreductibles. Pero no quiero entrar aquí en una discusión prolija sobre la filosofía de la ciencia en general.
¿Tengo razón al suponer que esta configuración de "túnel de relojes" ya está en movimiento, antes de que el gemelo viajero acelere para moverse con él (es decir, para embarcarse en el tramo de ida)? Adelante.
Sí, se supone que el túnel se ha estado moviendo inercialmente desde un tiempo anterior a cualquiera de los eventos que queremos utilizar con el propósito de asignar coordenadas de tiempo en el marco del tercer observador, y también se le aplicó de antemano el procedimiento de sincronización de Einstein ( tenga en cuenta que si ve la definición de simultaneidad del marco de la Tierra como metafísicamente correcta, entonces los relojes en este túnel están "realmente" desincronizados entre sí en términos metafísicos).
Entonces, digamos que cuando el gemelo viajero partía del gemelo de la Tierra, ambos estaban junto a un reloj de cuadrícula que decía t = 0.0 años. Luego, el gemelo viajero se aleja inercialmente de la Tierra y finalmente se da la vuelta cuando su propio reloj muestra que han pasado 4,0 años; cuando se da la vuelta, está junto al mismo reloj de cuadrícula que localmente también marca 4,0 años. Mientras tanto, cuando el propio reloj del gemelo de la Tierra muestra que han pasado 3,2 años, está junto a un reloj de cuadrícula diferente que localmente marca 4,0 años. Y cuando los gemelos se reencuentran, el reloj cuadriculado junto a ellos marca 12,5 años, el de los gemelos de la Tierra marca 10, el del viajero marca 8.
Tengo problemas para analizar esta situación. Parece decir que el gemelo viajero está por delante del gemelo de la Tierra en el cambio de dirección (4.0 frente a 3.2) en lugar de que cada uno afirme que el otro está detrás. En segundo lugar, ¿cuál es la longitud de zancada entre cada reloj en este túnel de rejilla? ¿Todos están de acuerdo con esa longitud? Si no están de acuerdo con la longitud de la zancada, entonces no pueden estar de acuerdo con la sincronización de todos los relojes en el túnel de rejilla.
¿Está preguntando sobre la longitud de zancada entre un par de relojes que mediría alguien moviéndose muy lentamente en relación con los relojes en la cuadrícula (es decir, la distancia en su marco de reposo), o está preguntando sobre la distancia entre los relojes en un momento dado? instante medido en el marco de la Tierra (¿que presumiblemente consideraría como la distancia 'verdadera' metafísicamente?) Debido al efecto de contracción de la longitud relativista (que también es una característica de la interpretación lorentziana), estas distancias serán diferentes.
"que presumiblemente consideraría como la distancia 'verdadera' metafísicamente" - no, en absoluto. Para ser franco, no tengo exactamente claro cómo razonar sobre el asunto: supongo que la contracción de la longitud también es mutua hasta cierto punto, en cuyo caso no puede ser real. Pero lo que hemos aclarado es que esta es solo una configuración más compleja que adolece de los mismos problemas de interpretación si los efectos observados son reales o no.
Claro, pero aparte de la cuestión de las distancias y solo centrándose en las mediciones de tiempo/simultaneidad, presumiblemente vería que las mediciones en esta cuadrícula no nos dan la simultaneidad "real" y las velocidades de reloj relativas, ¿no? De todos modos, estaba entrando en detalles porque antes dijo "pero no estamos de acuerdo en cuáles son esas lecturas locales". ¿Está diciendo que no está de acuerdo con lo que dije sobre cuáles serían las lecturas locales (que en la medida en que son predicciones sobre relojes físicos cerca uno del otro, deben predecirse en todos los marcos), o simplemente haciendo un comentario sobre la interpretación filosófica?
"pero aparte del problema de las distancias y solo centrándose en las mediciones de tiempo/simultaneidad" , no hay una división válida entre los problemas, porque qué celda de la cuadrícula es dónde y cuándo es crucial. "que en la medida en que son predicciones sobre relojes físicos cerca uno del otro, deben predecirse en todos los marcos" - no, no estoy seguro de que haya captado el problema de su propia configuración, porque si bien podemos estar de acuerdo en que el reloj de la Tierra es siempre cerca de un reloj de cuadrícula, la pregunta es cuál ya qué hora!
"o simplemente haciendo un punto acerca de la interpretación filosófica?" - seguramente esa ha sido la cuestión esencial todo el tiempo: la cuestión de si afirmaciones como "cada reloj funciona más lento que el otro" son efectos reales que reflejan afirmaciones válidas sobre los estados locales de los relojes, o si son meros efectos de señalización atribuibles a la dinámica de cómo se emite una señal y luego se recibe desde el espacio. Es decir, dinámicas que pueden variar independientemente de los estados locales de los emisores, y que pueden variar tanto por el movimiento del emisor como del receptor (o de ambos).
Más allá de lo que sugerí anteriormente, si esta comprensión mía puede llamarse "realismo" (específicamente porque insiste en el significado de los estados locales en este contexto, que existen independientemente de las señales que recibimos como evidencia de esos estados), la cuestión no es sólo que seas un "antirrealista" (según esta definición), sino que (a menos que estés jugando tus cartas de forma intencionada) no captas fácilmente la distinción que implica este realismo en comparación con tu propia filosofía, ni el hecho de que que el formalismo de la relatividad es totalmente compatible con ambos.
"No estoy seguro de que haya captado el problema de su propia configuración, porque si bien podemos estar de acuerdo en que el reloj de la Tierra siempre está cerca de un reloj de cuadrícula, la pregunta es cuál y a qué hora". Entiendo este punto, es por eso que dije en un comentario anterior 'si ve la definición de simultaneidad del marco de la Tierra como metafísicamente correcta, entonces los relojes en este túnel están "realmente" desincronizados entre sí en metafísica términos' . Dada una interpretación lorentziana, por supuesto, la comparación de eventos junto a diferentes relojes de cuadrícula no nos dice el tiempo "real" entre eventos.
Pero espero que esté de acuerdo en que si el reloj de la Tierra marca 0,0 cuando pasa justo al lado de un reloj de cuadrícula que marca 0,0, y el reloj de la Tierra marca 3,2 cuando pasa justo al lado de un reloj de cuadrícula diferente que marca 4,0, entonces, aunque esto no No significa que el reloj de la Tierra vaya más lento en ningún sentido "real", estos hechos locales siguen siendo realidades objetivas en sí mismos, no tiene sentido que los retrasos en las señales lleven a diferentes observadores a estar en desacuerdo sobre ellos (ya que las observaciones se pueden hacer con cámaras en los relojes de cuadrícula y la transmisión de imágenes, y todos verán las mismas imágenes en las imágenes).
'si esta comprensión mía puede llamarse "realismo" (específicamente porque insiste en el significado de los estados locales en este contexto) ¿Qué quiere decir exactamente con "significado"? Diría estados locales, como lo que leen dos relojes específicos. cuando pasan uno al lado del otro, son "significativos" en el sentido de que son hechos físicos objetivos e independientes del marco. Simplemente no creo que haya un hecho objetivo comprobable empíricamente sobre las comparaciones no locales del tiempo entre diff . eventos muy separados en el espacio y el tiempo, aunque podría haber una verdad metafísica no comprobable sobre esto.
"Pero espero que esté de acuerdo en que si el reloj de la Tierra marca 0,0 cuando pasa justo al lado de un reloj de cuadrícula que marca 0,0", ¡ eso plantea la pregunta! Para que exista tal medición, tendrá que manejar las señales. tanto del reloj local como de la cuadrícula en movimiento. La pregunta vuelve entonces, si recibe una lectura de cero de un reloj en la cuadrícula en movimiento, ¿dónde y cuándo se emitió esa señal de cero? Está suponiendo que debe emitirse junto a el reloj local en tiempo y lugar (porque estaban siendo conducidos muy de cerca). (1/2)
El problema es que un ciclo de la señal móvil no se puede sincronizar con la señal local, porque son de intervalos desproporcionados . Eso no se debe a que los relojes funcionen mal o tengan un diseño desproporcionado, sino a la dinámica de las ubicaciones cambiantes en las que se emite y recibe la señal. La relatividad cuantifica y corrige esa inconmensurabilidad permitiendo una comunicación sistemática, pero la pregunta entre nosotros es cómo surge esa inconmensurabilidad en primer lugar. (2/2)
Sospecho que básicamente pasa por alto el hecho de que los relojes (y las señales de ellos) deben medirse en intervalos y no en instantes (por lo que un reloj en movimiento debe medirse en una variedad de ubicaciones), y también que la sincronización depende de relojes que avanzan al mismo tiempo. intervalos Dicho de manera trivial, no se pueden sincronizar relojes que no marcan la misma hora. El problema de la relatividad (en cuanto a los efectos simétricos en el cronometraje, no los asimétricos) no es que los relojes no marquen la misma hora, es que la dinámica impide la sincronización de la señal entre el emisor y el receptor.
No estoy seguro de lo que quiere decir con "un ciclo" de la señal del reloj en movimiento "sincronizada" con la señal del reloj de la Tierra. Solo estoy hablando de los dos relojes que marcan el mismo tiempo de 0,0 años en una fotografía tomada en el momento de máxima aproximación cuando están separados por una distancia muy pequeña (digamos unos pocos centímetros). Supongo que se puede decir que cualquier fotodetector requiere un tiempo de exposición al menos tan largo como el período de los fotones que detecta, pero para la luz visible esto es menos de un picosegundo, no hay necesidad de asumir relojes con ese nivel de precisión en su lecturas
"Diría que los estados locales, como lo que leen dos relojes específicos cuando pasan uno al lado del otro, son 'significativos' en el sentido de que son hechos físicos objetivos e independientes del marco", pero no estás leyendo los estados, tú Estamos leyendo una señal (de velocidad limitada) que lleva información sobre esos estados. Por lo tanto, las peculiaridades de cómo se comporta la señal pueden tergiversar potencialmente los estados (incluidos los momentos exactos en los que prevaleció cualquier estado). (1/2)
Esa postulación de una señal que potencialmente puede ser modificada o manipulada independientemente del estado de su emisor, no es metafísica, es "realidad". Todavía tiene aspectos metafísicos en el sentido de que se postula la existencia misma de tales estados locales, cuando todo lo que podemos observar son señales que les conciernen, pero fuera de una minoría de la comunidad física que se obsesiona con las "observaciones", el resto del mundo no solo no tienen problema con la postulación metafísica del realismo, insisten en su uso porque así se describe una realidad objetiva. (2/2)
"Supongo que se puede decir que cualquier fotodetector requiere un tiempo de exposición al menos tan largo como el período de los fotones que detecta, pero para la luz visible esto es menos de un picosegundo, no hay necesidad de asumir relojes con ese nivel de precisión en sus lecturas". - mantenga ese pensamiento sobre el tiempo de exposición, porque lo que sucede cuando la foto lee 0 en el reloj local y múltiples segundos diferentes se ven borrosos en el reloj en movimiento (o simétricamente, múltiples en el local y 0 en el movimiento). Ahora la ubicación del reloj que muestra múltiples, ya no es segura. (1/2)
El punto que estoy tratando de ilustrar centrándome en un ciclo de la señal, es establecer que el hecho de que la señal tome un intervalo de tiempo para transmitir, significa que también (cuando se trata de un emisor en movimiento) también toma un intervalo de espacio para transmitir . (2/2)
'mantenga ese pensamiento sobre el tiempo de exposición, porque qué sucede cuando la foto lee 0 en el reloj local, y varios segundos diferentes se ven borrosos en el reloj en movimiento' Pero como dije, si el intervalo de tiempo mínimo del reloj no es inferior a un picosegundo, y el tiempo de exposición es más corto que el intervalo de tiempo mínimo que mide el reloj, no habría ninguna imagen borrosa para una foto fija tomada en frecuencias de luz visible. ¿Estás en desacuerdo con esto o simplemente te preocupa la pregunta ontológica de cómo podemos llamar "objetivos" a los eventos locales si no están definidos con una precisión infinita?
Pero cuanto mayor es la resolución del reloj que se observa, mayor es el margen en cuanto a dónde se puede ubicar la cuadrícula cuando los relojes marcan cero: hay una imprecisión de un segundo luz completo. No estoy argumentando que la inexactitud hace que la situación no sea objetiva; estoy enfatizando que es en estas mismas inexactitudes que surgen las diferentes observaciones en relatividad. Al tener un intervalo de reloj que es relativamente mayor que el intervalo de exposición, todo lo que hace es ampliar la ventana de posibles lugares (es decir, relojes en movimiento) desde donde se podría haber señalado el cero. (1/2).
Si lo compensa aumentando la velocidad del reloj mostrada (para que se muestren los picosegundos, no solo los segundos completos) para tratar de lograr la sincronización con la velocidad de exposición (es decir, una condición que crea la menor incertidumbre posible en cuanto a la ubicación), entonces regresa al problema de que la tasa de exposición no puede ser acorde con ambas frecuencias de reloj, y sin importar cuál optimice para la sincronización, eso deja un margen de imprecisión para el otro, y ese es el margen en el que la "otra" cámara en el lado de la cuadrícula , si se configura de forma completamente simétrica, reclamará un resultado diferente. (2/2)
Sin embargo, estoy seguro de que esta "cuadrícula" está complicando la discusión. El punto esencial es simplemente que no puedes sincronizar y mantener sincronizadas las señales de dos relojes idénticos que se mueven relativamente, porque los intervalos de las señales son diferentes por el movimiento. Si se están alejando, el no local siempre parecerá avanzar lentamente; si se está acercando, el no local parecerá acelerar. Y si uno acelera para invertir el curso, ninguno de los dos estará de acuerdo en cuanto a la longitud respectiva de las piernas, porque uno es local a la aceleración, el otro remoto.
"Pero cuanto mayor es la resolución del reloj que se observa, mayor es el margen en cuanto a dónde se puede ubicar la cuadrícula cuando los relojes marcan cero: hay una imprecisión de un segundo luz completo". No veo por qué debería ser cierto, ¿de dónde sacas ese número? ¿Qué está suponiendo para cosas como la velocidad relativa de los relojes (¿todavía 0.6c como en mi ejemplo?), la resolución de tiempo de los relojes y el tiempo de exposición necesario para que la cámara obtenga una imagen? Me parece que cualquier incertidumbre en la posición sería aproximadamente igual a la velocidad del reloj multiplicada por el tiempo de exposición.
La incertidumbre en la posición no es solo una función de la frecuencia del reloj y el tiempo de exposición, sino también de la cantidad de movimiento entre las fuentes. Aumentar la velocidad del reloj y reducir el tiempo de exposición no reduce la incertidumbre debida al movimiento, porque las cosas aumentan simétricamente en ambos lados (para preservar las condiciones locales comparables), por lo que la discrepancia entre los dos lados termina siendo la misma. - una señal de reloj aún termina siendo más rápida o más lenta por el mismo factor, y es el factor que cuenta para mantener (o romper) la sincronía.
¿Estás hablando de incertidumbre cuántica o de algún otro tipo? De cualquier manera, no veo por qué la frecuencia del reloj importaría en la mayoría de los casos, la posición de un reloj que se mueve a 0.6c no debería ser más incierta en una foto con un tiempo de exposición determinado que la posición de cualquier otro objeto moviéndose a 0.6c (una roca, digamos). La única forma en que importaría la resolución del reloj es si los dígitos cambian incluso más rápido que el tiempo de exposición, de lo contrario, solo obtendrá una imagen fija y no borrosa de una lectura particular en el reloj.
"La única forma en que importaría la resolución del reloj es si los dígitos cambian incluso más rápido que el tiempo de exposición", pero si los dígitos cambian más lento que el tiempo de exposición, entonces hay incertidumbre en cuanto al tiempo y la posición que es proporcional a la discrepancia. Supongamos que coloca rocas en lugar de relojes en la cuadrícula, es decir, los "dígitos" que se muestran no cambian en absoluto. ¿Qué ha demostrado al capturar la imagen? Nada, porque al mirar la foto no sabes cuándo se tomó la foto. No parece comprender la importancia de correlacionar los ticks con las exposiciones.
"¿Estás hablando de incertidumbre cuántica?" - no como tal, no, aunque como ese principio, esto se refiere a las limitaciones de un sistema basado en ondas.
'Digamos que coloca rocas en lugar de relojes en la cuadrícula, es decir, los "dígitos" mostrados no cambian en absoluto, ¿qué ha demostrado al capturar la imagen?' Estaba sugiriendo reemplazar el objeto en movimiento con una roca, no el reloj en el que está montada la cámara. Si una roca pasa volando por esa cámara, puede usar su propia marca de tiempo para asignar un tiempo a la roca que pasa junto a ella. Por supuesto, habrá cierta incertidumbre en el momento exacto en que ocurrió el pase cercano dependiendo de la resolución del reloj, pero usted sugirió incertidumbre de posición. Y esa incertidumbre temporal no depende de la velocidad de la roca.
"Si una roca pasa volando por esa cámara, puede usar su propia marca de tiempo para asignar una hora a la roca que pasa junto a ella". - ¿Te refieres a una marca de tiempo en la roca (más como los índices pintados en los postes de luz)? ¿O la marca de tiempo en la propia cámara? De cualquier manera, has derrotado el propósito de la cuadrícula. Estoy seguro de que mejores hombres que nosotros hemos luchado con este problema con más de 400 caracteres disponibles. Mi propósito aquí no es derrotar a la relatividad, sino explicar por qué un medio ondulatorio clásico ( cualquiera de esos medios, incluido el sonido) implica necesariamente los efectos que describe la relatividad. (1/2)
El propio Einstein tenía claro que la relatividad no era incompatible con un "éter" -es decir, un medio cuyo comportamiento se describe clásicamente-, simplemente establecía que el movimiento en ese medio no podía detectarse por ningún medio local (porque las leyes locales de la física son siempre lo mismo). Además, dado que las leyes locales de la física son siempre las mismas, no existe un marco preferido, y las leyes locales siempre se pueden describir completamente mediante el uso de un marco local. El problema con la señalización entre partes móviles es que es inherentemente no local: su propósito es mediar entre localidades. (2/2)
Y para aclarar mi terminología, una "localidad" es cualquier disposición de partes en las que nada se mueve entre sí. Cada vez que las cosas se mueven (y a pesar de la poca importancia de la relatividad a bajas velocidades), surgen efectos relativistas y esas partes ya no están todas en el mismo "marco local". Las leyes que gobiernan la interacción de las partes móviles son (tomadas como un todo) leyes no locales : la relatividad es el mapeo entre lo que es local y lo no local, y no existe un principio que diga un "marco preferido" para tal (que implican las interpretaciones de Lorentz) no existe.
¿Te refieres a una marca de tiempo en la roca (más como los índices pintados en los postes de luz)? ¿O la marca de tiempo de la propia cámara? En la Cámara. 'De cualquier manera, has frustrado el propósito de la rejilla'. ¿Cuál crees que es el propósito de la grilla? Creo que es solo para asignar coordenadas de tiempo a eventos en el marco de descanso de la cuadrícula, haciéndolo de una manera local que no depende de señales de larga distancia. Si los relojes están dispuestos a lo largo de las varillas de medición, como en el diagrama, podemos decir algo así como "la roca pasó la marca de 5 metros cuando el reloj de la cuadrícula marcaba 0,000352 segundos".
Y funciona igual si el objeto que se mueve en relación con la cuadrícula es en sí mismo un reloj, entonces puedes decir algo como "el reloj A pasó la marca de 5 metros cuando el reloj de la cuadrícula marcaba 0,000352 segundos y el reloj A marcaba 0,000071 segundos". 'es para explicar por qué un medio ondulatorio clásico (cualquiera de esos medios, incluido el sonido) implica necesariamente los efectos que describe la relatividad.' No todos los efectos, seguramente? No es necesario que haya ninguna dilatación del tiempo de los relojes en movimiento en la física clásica, por ejemplo, en un escenario de tipo paradoja gemela, ambos relojes permanecerían sincronizados cuando se reúnan y comparen los tiempos localmente.
"¿Cuál crees que es el propósito de la rejilla?" - El propósito de la cuadrícula, según entendí, era tratar de eliminar el "espacio de aire" entre dos relojes que no están estacionarios entre sí. Dado que el gemelo viajero está a eones de distancia, si nosotros (es decir, usted) introducimos una cuadrícula de relojes, entonces establecemos una cadena de sincronización hasta la Tierra, y los relojes en las cercanías de la Tierra representan de manera equivalente la lectura en el reloj llevado por el gemelo viajero. (1/2)
En mi opinión, el problema es que la cuadrícula no solo introduce una complejidad innecesaria en la configuración (quizás, la confusión que esto genera es cómo el dispositivo evita cumplir con su destino), sino que tan pronto como se construye y se mueve, ya toma por sentado las mismas cosas en cuestión. La sincronización de la cuadrícula presupone una longitud de zancada conocida entre cada reloj, ya que no se pueden sincronizar relojes mediante señales no instantáneas, si se cuestiona la distancia que los separa. Pero dado que ambas partes no están de acuerdo en esa longitud de zancada, el esquema se impugna por falta de sincronía acordada. (2/2)
"podemos decir algo como 'la roca pasó la marca de los 5 metros cuando el reloj de cuadrícula marcaba 0,000352 segundos'; esto también presupone lo mismo en cuestión, porque fue en la marca de los 5 metros donde se emitió la señal del reloj , ¿o se emitió más atrás hacia la marca de 4 metros? Si todo estuviera estacionario, la pregunta sería absurda, pero dado que la cuadrícula se mueve en relación con la cámara, en realidad es necesario que dondequiera que mire la cámara, no sea el punto desde el que la señal es emitida por la rejilla, porque la rejilla emite la señal en un rango de espacio (1/2)
Es decir, mientras que normalmente miramos la imagen de una cámara y extrapolamos líneas rectas a un punto en la profundidad de campo (de acuerdo con los principios ópticos normales) para argumentar que dondequiera que mire la cámara, es el lugar donde se originó la señal, que deja de ser el caso cuando se trata de un emisor de ondas en movimiento. No se origina en ese punto porque la onda se emite durante un intervalo de tiempo, y si el emisor se mueve en el espacio, la emisión se produce en varios lugares del espacio y del tiempo. Ya no es válido decir que sucedió frente a la cámara. 2/2
"¿No todos los efectos, seguramente? No es necesario que haya ninguna dilatación del tiempo de los relojes en movimiento en la física clásica" - ¡sí! Ocurre en todos los medios de onda, si implementa el reloj en términos de un intercambio local de señales a través del medio de onda (es decir, la analogía de un "reloj de luz" con un elemento resonante que pasa continuamente de un lado a otro a través del medio) . Si implementa un "reloj de sonido" para mantener el tiempo, obtiene la dilatación asimétrica del tiempo del gemelo en movimiento cuantificada por la relatividad, pero por razones de Lorentzian (que la velocidad del sonido no es isotrópica en todas las direcciones). (1/2)
Obviamente, la analogía con el sonido no es total en todos los aspectos, pero puede reproducir claramente los elementos de dilatación del tiempo, cuando los relojes empleados se ven obligados a emplear el medio en cuestión como parte de su funcionamiento normal. En la analogía del sonido, ambos lados no solo afirman que el otro está perdiendo tiempo en el tramo de ida y recuperándolo en el de regreso, sino que también están en desacuerdo sobre la longitud de los tramos, y uno realmente regresa "más joven" con un reloj que tiene progresado menos. (2/2)
El principal obstáculo entre muchos físicos, en mi opinión, es su insistencia axiomática en que las velocidades de los relojes atómicos no varían. Hablando en términos prácticos, la evidencia experimental demuestra que los relojes atómicos varían con el movimiento, pero esto se desvanece con la explicación de que "los relojes tomaron un camino de diferente longitud a través del espacio-tiempo", a pesar de que esa descripción (sin su jerga infeliz) es equivalente a la declaración de que "el reloj se movió a través del espacio y su ritmo se atrasó".
'esto también presupone lo mismo en cuestión, porque ¿fue en la marca de 5 metros donde se emitió la señal del reloj, o se emitió más atrás, hacia la marca de 4 metros?' ¿De qué "señal del reloj" estás hablando? Estás respondiendo a un escenario en el que una cámara que está montada en el reloj toma una fotografía de una roca cuando pasa a una distancia insignificantemente pequeña de la cámara, con la cámara conectada al reloj de tal manera que el reloj está leyendo en ese momento. la foto se toma aparece como una marca de tiempo. El reloj simplemente se montaría de forma permanente en la marca de 5 metros.
'Si implementa un "reloj de sonido" para mantener el tiempo, obtiene la dilatación de tiempo asimétrica del gemelo en movimiento cuantificada por la relatividad' Obtendría una desaceleración de un reloj de sonido (uno basado en el rebote de ondas de sonido de un lado a otro entre reflectores para servir como garrapatas), aunque no creo que la ecuación de dilatación del tiempo real sea la misma. Pero en la física clásica no relativista es posible construir otros tipos de relojes (relojes mecánicos) que no se dilaten. En relatividad, los efectos se aplican a todos los relojes, lo que hace imposible determinar experimentalmente el marco de reposo de cualquier 'medio'.
Estoy confundido acerca de qué lado están las marcas del medidor. ¿Están del lado de casa o del lado de la parrilla? No puedo ver el punto de tenerlos en el lado de casa (es decir, el mismo lado que la cámara que rompe la roca). En verdad, el escenario ha sufrido tantas modificaciones ahora, con elementos ahora tan diversos como rocas e imágenes con marcas de tiempo, y sin diagramas de referencia, que no estoy seguro de que todavía estemos cantando con la misma partitura. (1/2)
Tenga en cuenta que mi argumento aquí no es contra la relatividad o cualquiera de sus predicciones. Fue para enfatizar cómo algunos de los efectos que describe surgen directamente de la naturaleza de un medio de señalización basado en ondas. (2/2)
"Pero en la física clásica no relativista es posible construir otros tipos de relojes (relojes mecánicos) que no se dilaten. En la relatividad, los efectos se aplican a todos los relojes, lo que hace imposible determinar experimentalmente el marco de reposo de cualquier 'medio'. " - pero ese era precisamente el problema que tenía Lorentz, ¿no? ¿Estamos tratando con un medio clásico que es simplemente indetectable (al menos por cualquier experimento concebido hasta ahora), o no estamos tratando con tal medio? (1/2)
La evidencia científica no determina la pregunta; la única distinción es si asumimos la existencia de ese medio (dado que su presencia es capaz de explicar completamente los efectos que vemos, es consistente por analogía con el funcionamiento que vemos en otros sistemas físicos donde la presencia del medio es verificable independientemente, y la relatividad es totalmente consistente con ello), o postulamos (sin estar obligados a hacerlo por ninguna evidencia científica) algún tipo de filosofía física especial que no tiene otra aplicación conocida y da "justo así" respuestas a todas las preguntas que se le hacen? (2/2)
En resumen, mientras que la distinción entre el sonido y la luz es que tenemos alternativas a los relojes de sonido, pero no tenemos alternativas a los relojes de luz, pero los dos se pueden describir en términos generales por el mismo marco conceptual y parecen tener las mismas características, entonces mi primer recurso es simplemente aceptar la indetectabilidad de un medio ligero. No es embarcarme en un vuelo filosófico de la fantasía acerca de cómo puedo dar sentido a la situación mediante una variedad de postulados no clásicos que son exclusivos de esta área de la ciencia .
El hecho de que no se sepa que el medio ligero es detectable no es un problema importante para mí, porque no es una extravagancia filosófica: es una reutilización ordenada y sin complicaciones de conceptos familiares que surgen del análisis de sistemas basados ​​​​en ondas donde el medio es detectable independientemente. No tengo muy claro por qué debería ser detectable de forma independiente antes de que los físicos lo tomen en serio, en lugar de que su existencia simplemente se deduzca por su poder explicativo y por la existencia de sistemas naturales similares que son tratables por la ciencia .
Las marcas de medidor en el lado de la cuadrícula siempre se utilizan para asignar coordenadas de posición en el marco de la cuadrícula. No creo que agregar una suposición de marcas de tiempo realmente aumente la complejidad, igualmente podría imaginar que la cámara montada en el reloj de cuadrícula está colocada de tal manera que la imagen muestra la lectura en el reloj en el que está montado junto con el objeto (ya sea una roca o un reloj sin cuadrícula) que pasa junto a él. El punto es que la posición y las coordenadas de tiempo del evento de algo que pasa están determinadas por un reloj local y una vara de medir en reposo en relación con la cámara.
Pero ese era precisamente el problema que tenía Lorentz, ¿no? ¿Estamos tratando con un medio clásico que es simplemente indetectable (al menos por cualquier experimento concebido hasta ahora), o no estamos tratando con tal medio?' He dicho algunas veces en esta discusión que no tengo ningún problema con una interpretación lorentziana siempre que se entienda como una suposición filosófica que no es comprobable. Pero creo que una interpretación filosófica alternativa basada en una suposición ontológica de una geometría de espacio-tiempo 4D sin un presente preferido (lo que los filósofos llaman la 'teoría B del tiempo') también es coherente.
No estoy criticando el uso de la cuadrícula per se (como una táctica), simplemente digo que ahora se ha modificado hasta el punto en que ya no estoy seguro de tener una imagen coherente de sus elementos, los roles que desempeñan. están realizando, las diferentes mediciones realizadas dentro de cada marco, o incluso qué (y de quién) afirma que se supone que establece o refuta la configuración. Creo que si vamos a seguir adelante, toda la situación tendría que ser reconstruida de forma coherente con diagramas de apoyo y los argumentos ensayados desde la base (a lo que este formato no se presta).
"He dicho varias veces en esta discusión que no tengo ningún problema con una interpretación lorentziana siempre que se entienda como una suposición filosófica que no es comprobable". - acordado. La interpretación es lo único que está en juego aquí. Debo agregar que no es menos comprobable que la interpretación que defiendes.
Mi pregunta original se refiere al hecho de que, para que su interpretación sea sostenible, la constancia de los relojes atómicos debe postularse axiomáticamente. Además, se debe proponer algún tipo de lugar en el tiempo al que vayan las cosas, un lugar que se defina individualmente para cada diferencia de velocidad posible entre dos cosas y que permita que cada reloj esté detrás del otro (un concepto que realmente no puedo). no tiene sentido). Pregunté en términos, ¿cuál es la justificación de esto, cuando el enfoque lorentziano servirá (sin rarezas, sin pérdida de objetividad e idéntico poder predictivo)?
Entonces, en mi opinión, esta teoría B del tiempo es completamente antifísica y extravagante. Parece tener sentido en sus propios términos puramente en el ámbito de las cosas del pensamiento, pero postula una cuarta dimensión independiente, movimiento en el que se presume que es detectado y medido por relojes, pero esa independencia no está respaldada por la realidad práctica de cómo funcionan los relojes. Y el resultado neto, el poder predictivo neto, es el mismo que si lo describiera todo en términos de Lorentz.

Respuestas (5)

Creo que veo el corazón de tu pregunta. De hecho, no tiene nada que ver con la relatividad. Permítanme intentar reformular.

En el pasado (digamos 1700) teníamos relojes de péndulo para medir el tiempo. Dijimos que cada tictac del reloj era un segundo. Sin embargo, los marineros de la época se dieron cuenta de que si colocaba un reloj de péndulo en un barco, este funcionaría "rápido" o "lento" porque el balanceo del barco o las variaciones de temperatura alterarían la física del péndulo. ¿Cómo podían saber que iba rápido o lento? Podrían llevar el reloj de vuelta a Greenwich, donde estaba originalmente configurado, y notar que su reloj marcaba 100 000 veces, mientras que el reloj de Greenwich marcaba 120 000 veces. Esto se explica fácilmente por lo que le sucedió al reloj que pertenecía a los marineros en el barco.

Ahora, su preocupación es que cuando la gente habla de relojes atómicos (el "estándar" moderno para el cronometraje) no mencionan efectos nocivos como el "balanceo del barco" que puede hacer que los relojes atómicos funcionen más rápido o más lento. Su preocupación es que el reloj atómico podría estar "meciéndose", pero simplemente lo escondemos debajo de la alfombra y decimos que el "tiempo" corre rápido o lento. La pregunta es ¿por qué el cambio de actitud? Previamente, reconocimos los mecanismos físicos que podrían alterar la forma en que funciona el reloj y admitimos que empeoran el reloj, pero ahora simplemente lo ocultamos diciendo que el tiempo corre rápido o lento. ¿Lo que da?

Espero que lo anterior haya sido una reafirmación precisa de su pregunta. Permítanme ahora dar mi respuesta.

1) Primero, el título de este post es "¿Qué miden los relojes?". Has sugerido que los relojes miden tasas. Creo que esto es incorrecto. Creo que los relojes miden un NÚMERO de eventos. Un reloj de péndulo mide cuántas veces el péndulo llega a su extremo derecho. Un oscilador de cuarzo mide cuántas veces sus puntas alcanzan los extremos de su movimiento. Un reloj atómico mide cuántas veces la función de onda del electrón gira alrededor del núcleo*. ¿Qué son las tarifas entonces? Bueno, hemos definido que el segundo es algo así como: Cada vez que el reloj de cesio en Boulder marca 9,192,631,770 veces, decimos que ha pasado un segundo. Por lo tanto, ahora podemos decir (basándonos en la definición) que el reloj de cesio funciona a una velocidad de 9 192 631 770 tics por segundo. La cantidad medida fundamental es un número de eventos, la cantidad definida es un tiempo,

2) Bien. Pero, al igual que en el barco, ¿los efectos nocivos no pueden afectar la rapidez con la que funciona el reloj de cesio? Es decir, la cantidad de "tiempo" que transcurre entre dos tics podría cambiar si el reloj de cesio está "oscilando". ¿Cómo es que no escucho sobre ese tipo de cosas? Bueno, probablemente no haya oído hablar de ese tipo de cosas porque no está inmerso en el campo de la medición de precisión o la física atómica. Los físicos atómicos, de hecho, se preocupan todo el tiempo por las cosas que alteran el ritmo de sus átomos. Las cosas que pueden estropear la tasa de tictac son los campos eléctricos/magnéticos (hacen que los átomos tictac más rápido o más lento), las colisiones con otros átomos, etc. Un problema con los relojes atómicos es que la radiación de cuerpo negro emitida por la cámara de vacío a temperatura ambiente en la que residen los átomos hace que los átomos cambien su frecuencia de tictac. Por todo esto se reconoce que el segundo se define como la cantidad de tiempo que tardaría un átomo de Cesio en marcar 9.192.631.770 veces si estuviera a 0 K, en 0 campo magnético, en 0 campo eléctrico sin influencias externas. Sin embargo, los físicos se dan cuenta de que esto es imposible de lograr en el laboratorio. No obstante, hay beneficios tecnológicos al intentar hacer lo mejor que pueden. Así que realizan un cierto experimento y miden las garrapatas de cesio de una manera particular lo mejor que pueden e informan al mundo cada vez que su cesio garrapatas.

3) Si los relojes siempre pueden tener algún error, ¿cuál es el beneficio de tener relojes? Bueno, aunque los relojes siempre están mal hasta cierto punto, también están bien hasta cierto punto. Por ejemplo, mi amigo puede decirme: "¡Oye, nos vemos en la bolera después de que el oscilador de cuarzo de MI reloj marque 34 875 329 veces!" e incluso si él va a su casa (que mantiene a 65 F) y yo voy a mi casa (que mantengo a 70 F) y dejo caer MI reloj en el fregadero (es resistente al agua) todavía puedo sacarlo y tener fe en que una vez que mi reloj marque 34,875,329 veces, el reloj de mi amigo TAMBIÉN habrá marcado la misma cantidad para que pueda llegar a la bolera y no molestarlo con 5,328 tictac de retraso.**

El objetivo de fabricar relojes cada vez mejores es que los humanos puedan tener tanta fe en los dispositivos de mantenimiento del tiempo de los demás en escalas de tiempo cada vez más finas. Por ejemplo, si los físicos del reloj atómico de Boulder, Colorado, hacen un buen trabajo manteniendo el reloj en marcha (con campos magnéticos mínimos, etc.) y los físicos del reloj atómico de París, Francia, hacen un buen trabajo con el reloj, entonces las dos partes pueden tener fe en que, incluso después del paso de un largo período de tiempo*****, seguirán contando el mismo número de tictac de sus relojes. Esto tiene implicaciones prácticas cuando esos relojes se utilizan para sincronizar diferentes relojes en todo el mundo, incluidos los que se utilizan para el GPS satelital y el funcionamiento de los mercados bursátiles mundiales, los cuales se basan en la medición de diferencias muy pequeñas en el tiempo.

4) Y otra nota que nos recuerda a los marineros. Preguntemos de nuevo cómo sabían los marineros que su reloj iba rápido o lento (aparte de ver a sus compañeros de tripulación patear el péndulo unas cuantas veces). Notarían que el sol no saldría cuando esperaban según su reloj o llevarían su reloj a Greenwich y lo compararían allí. En ambos casos están comparando su reloj con otro fenómeno físico oscilatorio. La clave es que están comparando su reloj con un fenómeno que es más "estable"*** que el de su nave. Sin embargo, estos dos relojes también son, por supuesto, susceptibles a las fluctuaciones del reloj. Si la temperatura cambia en Greenwich, eso también afectaría su reloj de péndulo, pero no tanto como el péndulo más pequeño en el barco acosado por el duro entorno marítimo.

Además de la estabilidad, es importante que un reloj estándar se pueda recrear en otro lugar y proporcione los mismos resultados. Eso se ilustra con la presencia de relojes atómicos Cs similares en todo el mundo. La belleza es que, si puedes controlar el entorno lo suficientemente bien, un átomo de Cs en Boulder tiene la misma frecuencia de tic-tac que un átomo de Cs en París. Si todos pueden sincronizarse con estos y otros relojes en el sistema internacional de relojes atómicos, entonces podemos estar seguros de que todos podemos estar de acuerdo en el tiempo para participar en 10 dieciséis más o menos y esto puede ser útil tecnológicamente. Sin embargo, como ha identificado, HAY un límite en la precisión que podemos tener. Esto es conocido y reconocido y la gente siempre está trabajando para mejorar este límite.

editar: una nota más aquí. Dado que los relojes miden el número de eventos que suceden, y el tiempo se deriva de esa medida, en cierto sentido, si el reloj de Greenwich se ralentiza o el reloj atómico de Boulder se ralentiza, es correcto decir que el tiempo mismo se ralentiza porque así es el tiempo. se define. Sin embargo, tiene razón al señalar que debemos reconocer que esto sucede debido a efectos físicos indeseables en nuestro aparato. Por eso reconocemos que estos relojes solo tienen un nivel finito de precisión y reconocemos cierto nivel de incertidumbre en la definición/medida del tiempo. Construir un reloj mejor significa reducir esta incertidumbre.

5) Hay una dimensión de su pregunta que implica relatividad especial, pero creo que, de hecho, es el punto menos interesante. En cierto sentido, podemos decir que los efectos relativistas son solo otro efecto externo que hace que el reloj funcione de manera diferente al reloj de Boulder. ¿Qué pasa si el reloj en Boulder está experimentando efectos relativistas especiales? Bueno, todavía podemos compararlo con el reloj de París y obtener buenos resultados con cierta precisión. Eventualmente, para construir un mejor reloj, tal vez sea necesario controlar tales efectos. Algunos efectos que limitan los relojes atómicos (hacen que funcionen de manera diferente) ahora son: colisiones atómicas, radiación de cuerpo negro, los láseres utilizados para medir los átomos, campos magnéticos dispersos, etc. Alguien más cercano al campo del reloj atómico podría hacer un mejor trabajo que yo en elaboración de esta lista.

6) Recomiendo leer el libro de divulgación científica/historia "Longitud" de Dava Sobel sobre la necesidad y la invención de cronómetros de precisión para la navegación naval en el siglo XVIII para comprender las razones prácticas POR LAS QUE queremos un reloj preciso y lo que necesitamos. decir por un reloj preciso. Quizás después de comprender algunas situaciones concretas del mundo real, tendrá una mejor visión de algunas de sus preguntas abstractas.

edit2: 7) Nota importante sobre la estabilidad del reloj. Cuando digo que un reloj es estable, ¿a qué me refiero? Bueno, digamos que tengo dos relojes de pulsera construidos que fueron fabricados uno tras otro en la línea de montaje. Si los sincronizo hoy, puedo observarlos durante un año y ver qué tan lejos están. Si bajan 30 segundos en un año, entonces puedo calcular una discrepancia fraccionaria.

30  s 1  año 10 6
No sé qué reloj es más correcto (preciso), pero sé que están de acuerdo con una parte por millón. Es decir, tienen una estabilidad relativa de 10 6 . Ahora los relojes atómicos Cs estándar son buenos para una parte en 10 dieciséis más o menos si se comparan entre sí**** Nuevamente, no sabemos qué reloj es más correcto, pero podemos decir que los relojes atómicos son más estables que los relojes de pulsera porque pueden estar de acuerdo entre sí por una cantidad mayor de tiempo. tiempo

*Esto es un poco de física atómica aquí, pero los átomos usados ​​en los relojes atómicos pueden considerarse exactamente como péndulos. Es un sistema que oscila físicamente en el espacio. Esto puede ser tema para otra pregunta.

**Aunque, como todos sabemos, tener un reloj preciso no es garantía de que uno no llegue tarde. ¡Tal garantía requiere, además, una cierta responsabilidad personal!

***Donde la estabilidad se puede definir en un sentido técnico comparando el ritmo de tictac de un reloj con otro que está más controlado físicamente o comparando el ritmo de tictac de dos relojes si no hay un reloj "mejor". Ver apartado 7)

**** Los relojes atómicos más estables reportados de hecho usan Sr y tienen una precisión de una parte en 10 18 más o menos, pero estos no se utilizan como el estándar de tiempo oficial. Quizá en el futuro lo sean.

***** Tenga en cuenta que estos relojes marcan casi 10 10 (diez billones) de veces por segundo. En una estabilidad de 10 dieciséis estos relojes pueden funcionar durante más de 10 días y no fallar ni un tictac.

Ciertamente estoy convencido de que ha entendido el contexto de mi pregunta, ¡lo ha reafirmado bien! Como dices, los marineros en el mar no dijeron "el tiempo había ido más lento/más rápido en alta mar"! Entonces, ¿por qué algunos dicen que un reloj atómico enviado alrededor de la Tierra en un avión ha experimentado "dilatación del tiempo", en lugar de simplemente "balanceo del barco"? De hecho, ¿por qué algunos insisten en que el reloj atómico es estable? Sí, se puede decir que es estable excepto por la gravedad y el movimiento, pero entonces no es estable en absoluto : lejos de ser una constante física , fundamentalmente y en principio continúa siendo variable.
Bueno, los relojes atómicos en Boulder/Paris son estables porque no están experimentando ningún efecto relativista especial o general que cambie en función del tiempo, o si lo están, esos efectos cambian ambos relojes por igual, por lo que para todas las actividades terrestres podemos considerar el relojes para ser buenas referencias.
Con respecto al reloj atómico entrando en órbita y experimentando la "dilatación del tiempo". ¿Deberíamos considerar eso como "mecer el barco"? Bueno, en mi punto 5) dije que, en cierto sentido, podemos pensar en los efectos relativistas especiales como otro mecanismo externo para cambiar la velocidad del reloj RELATIVA al reloj de Boulder. Lo que quiere decir, sí, los efectos relativistas son solo otras formas de balanceo del barco. La parte importante es que podemos predecir el efecto de estos efectos relativistas en el tiempo de los relojes y vemos un buen acuerdo con nuestros modelos.
Sin embargo, como han dicho otros, bajo los efectos relativistas, TODOS los sistemas físicos sufrirán tal "balanceo" que se manifiesta como relojes que corren más rápido o más lento en relación con un reloj en un marco de referencia diferente. Dado esto, está bien decir que el "balanceo del barco" experimentado por un reloj atómico en una nave espacial es equivalente a la desaceleración del tiempo en la nave espacial en relación con el reloj en la tierra.
También acabo de editar la respuesta para agregar una sección 7) con un poco más de detalle sobre lo que quiero decir con "estable".
En lo que a mí respecta, estamos totalmente de acuerdo en los puntos que importan. Has captado lo que la mayoría de los demás no han podido, y puedo decir que entendemos la realidad física de la misma manera. Mi único punto es que no está bien equiparar "el balanceo del barco" con "el tiempo se ralentiza", porque eso lleva a la gente a razonar erróneamente acerca de que los procesos atómicos son una categoría especial de la física que no está sujeta a la variación ambiental. Es una posición completamente diferente decir que algo no está sujeto a variación, que decir que está sujeto a variación junto con todo lo demás .
Como lo ha dicho en este comentario, su punto es más pedagógico/semántico que cualquier otra cosa. Le preocupa que el lenguaje de la desaceleración del tiempo lleve a la gente a creer que los procesos atómicos no están sujetos a perturbaciones ambientales. Veo tu punto, pero no estoy de acuerdo con tu conclusión. Es posible creer que la relatividad especial en realidad tiene "el tiempo ralentizándose", pero también darse cuenta de que los átomos pueden estar sujetos a perturbaciones ambientales.
Lo que tal vez debería enfatizarse (como lo he hecho en mi respuesta) para aliviar su preocupación por los conceptos erróneos que surgen en otros es el hecho de que, de hecho, los átomos ESTÁN sujetos a perturbaciones ambientales. Creo que lo que sucede más probablemente que lo que estás describiendo es que las personas aprenden que los relojes atómicos mantienen el "tiempo perfecto" o que los átomos son osciladores "prístinos". Luego asumen que los átomos son sistemas físicos verdaderamente perfectos (esto debería ser una señal de alerta) y extienden este concepto a la relatividad especial pensando que los relojes atómicos son medidas perfectas del tiempo y, por lo tanto, el reloj atómico funciona lento.
significa que el tiempo corre lento. Aquí es donde entras con preocupaciones sobre lo que miden los relojes (incluso los relojes atómicos). De hecho, le preocupa precisamente el argumento de que "dado que los relojes atómicos funcionan lentos bajo la relatividad especial, eso significa que el tiempo funciona lento". De hecho, el argumento de la relatividad especial es más directo. Dice que debido a que la velocidad de la luz es constante en todos los marcos, debe ocurrir la dilatación del tiempo. No tiene nada que ver con los relojes atómicos. Los relojes atómicos son solo una corroboración experimental de la teoría. De hecho, confirma la teoría hasta la precisión de los relojes, 10 dieciséis
Para concluir y llegar al punto de su comentario, todas las partes deben apaciguarse al afirmar que A) las leyes de la relatividad especial o la física no otorgan un estatus privilegiado a los sistemas atómicos. Todos los sistemas físicos experimentan la dilatación del tiempo por igual. Por eso está bien decir que el tiempo se ralentiza en los marcos de referencia en movimiento. Y B) A pesar de posibles conceptos erróneos, los relojes atómicos, como todos los relojes, no son perfectos y están sujetos a perturbaciones que pueden hacer que funcionen más rápido o más lento. Estas perturbaciones pueden incluir efectos relativistas.
Por cierto, estoy un poco emocionado con este tema, así que sigo escribiendo. Echa un vistazo a este experimento en el que se ejecutaron dos relojes atómicos con uno elevado 1 metro. Experimentaron un desplazamiento hacia el rojo gravitacional relativo que hizo que funcionaran a diferentes frecuencias. Eso significa que para que estos relojes coincidan, no solo debe mantenerlos a la misma temperatura, campo magnético, etc., sino que también debe asegurarse de que estén en el mismo campo gravitatorio, de lo contrario, funcionarán de manera diferente. Esto le muestra lo difícil que es hacer un buen reloj en el 10 18 ¡nivel!
nist.gov/news-events/news/2010/09/… CW Chou, DB Hume, T. Rosenband y DJ Wineland. Relojes ópticos y relatividad. Ciencia. 24 de septiembre de 2010.
Ja ja. Ciertamente estás en el camino correcto conmigo. ¡Estoy planteando estas preguntas principalmente porque otros me transmitieron conceptos erróneos sobre la física ! Y uno solo tiene que revisar las respuestas aquí para ver que muchas personas no entienden de lo que están hablando. Mi preocupación más general es lo que dije, que argumentar que la gravedad o el movimiento afectan la velocidad de todos los procesos tiene un significado diferente de decir que los procesos atómicos son constantes y no se ven afectados por nada y que, en relatividad, es el paso del tiempo lo que es. afectado por el movimiento y la gravedad. (1/2)
No obtienes la misma calidad de razonamiento físico cuando las personas atribuyen la variable al tiempo mismo. Por ejemplo, no dicen "sí, la velocidad de un reloj atómico está sujeta a una variable ambiental: el paso del tiempo", porque concebir el "paso del tiempo" como una variable ambiental toca una fibra sensible en la metafísica de las personas. comprensión del concepto, y tienes que tratar de concebir cómo el tiempo "fluye" a través de todas estas cosas, y cómo el movimiento o la gravedad afectan este flujo. (2/2)
... La verdad, por supuesto, es que no hay flujo físico en absoluto: todo el concepto del paso del tiempo es metafísico.

El parámetro tiempo es un concepto derivado por abstracción de las observaciones físicas. Estas observaciones muestran que diferentes procesos físicos (en un lugar) exhiben una coherencia que también puede repetirse. Por ejemplo, el número de oscilaciones de un péndulo o de un volante corresponde repetidamente a una buena aproximación al vaciado de un reloj de arena o agua o al paso diario de una estrella debido a la rotación de la tierra. Esta coherencia depende, por supuesto, de la precisión de los dispositivos o procesos utilizados, pero la abstracción funciona bastante bien de que existe un parámetro. t llamado tiempo que mantiene una relación duradera y repetida entre diferentes procesos físicos. Usar el concepto de "tasa" para un proceso es solo una forma diferente de usar el concepto de tiempo porque la tasa es solo la cantidad de cambio dividida por el tiempo. Este concepto de un parámetro existente t que rigen todos los procesos físicos en un lugar ha sido extraordinariamente exitoso y se ha confirmado en todos los avances en la precisión de las implementaciones técnicas para representar este parámetro.

En mi opinión, probablemente no ha habido un cambio en el razonamiento con respecto a la posible influencia de varias condiciones físicas y errores en los dispositivos de cronometraje desde el siglo XIX. Y el concepto abstracto de tiempo independiente de los dispositivos de cronometraje existe desde la antigüedad. Sólo la comprensión de los procesos físicos utilizados y la reducción de errores han aumentado enormemente. Esto incluye la discontinuidad del concepto de un tiempo absoluto y la influencia de la gravedad sobre él en la teoría especial y general de la relatividad.

RE "probablemente no ha habido un cambio en el razonamiento con respecto a la posible influencia de varias condiciones físicas y errores en los dispositivos de cronometraje desde el siglo XIX"; ¿No considera que la aceptación generalizada de la teoría de la relatividad de Einstein (/Lorentz) en el siglo XX sea tal cambio?
@ThePhoton: por supuesto, es un cambio en el concepto de que el tiempo no es absoluto y está influenciado por un campo gravitatorio, que mencioné. Pero esto probablemente no haya sido un cambio en los dispositivos de cronometraje .
Bien, específicamente sobre cómo razonamos sobre los dispositivos de cronometraje, señalaré que la varianza de Allan , una de las herramientas clave para medir los errores de cronometraje, solo se concibió en la segunda mitad del siglo XX.
Gracias por la respuesta @Freecharly. No estoy cuestionando el uso del tiempo (¡no veo que podamos prescindir de él!), estoy examinando lo que realmente es , y si los conceptos de tiempo y tasa a menudo se combinan de manera descuidada. Si "la tasa es la cantidad de cambio dividida por el tiempo", entonces es claramente relevante reconocer que lo que se mide es la tasa, no el tiempo. Son dos cosas diferentes (una incorporando a la otra), de las que se debe hablar de manera diferente (o al menos, se deben reconocer como cosas diferentes que solo pueden combinarse en contextos específicos).
... Cuando hablo de efectos relativistas, por ejemplo, mi argumento es que no se pueden combinar, porque el movimiento y la gravedad son variables en la medida de la velocidad. Sin embargo, claramente por los comentarios que he recibido, la gente afirma que los procesos atómicos tienen una tasa constante; no reconocen el movimiento y la gravedad como variables que afectan la tasa (al igual que la temperatura afecta a un péndulo), pero no pueden (de manera válida y rigurosa). ) se dice que afectan el tiempo .
...Tampoco ayuda redefinir el tiempo como equivalente a la tasa, porque ya hemos establecido que son dos conceptos diferentes, y la física aún tendría el desafío de explicar qué es el "tiempo", si no es un concepto abstracto y potencialmente metafísico. que se postula para que la teoría funcione. De hecho, ¡parece ser simplemente el valor residual, siempre que una "tasa" se divide por una constante! Y funciona sólo cuando la constante definida corresponde a una variable física sin cambios . La constante se vuelve inválida para derivar el "tiempo" de la tasa, si la variable física varía.
@Steve: la pregunta "qué hora es realmente" es, por supuesto, muy intrigante. Al final, el concepto de "tasa" tampoco puede prescindir del tiempo, la diferencia es solo lo que significa "cantidad de cambio". Para determinar la tasa de un proceso, necesita un proceso de referencia. Y, por supuesto, puede utilizar un proceso de referencia específico y compararlo con el investigado. Luego, tiene una tasa definida por el cambio del proceso 1 dividido por el cambio del proceso 2. Simplemente no usa la palabra tiempo porque no abstrae el concepto de muchos procesos de referencia, incluida la consideración de errores.
@freecharly, estoy de acuerdo en que la tasa no puede prescindir del tiempo, pero no lo hace idéntico al tiempo. Tengo la intención de formular otra pregunta eventualmente (continuando desglosando lo que dije en mi pregunta vinculada original), explorando que el tiempo no es una dimensión física independiente (como las 3 dimensiones espaciales son grados de libertad independientes), pero es en sí mismo una medida derivada de cambio en las dimensiones espaciales (es decir, una medida de tiempo puro no tiene sentido físico, es un concepto metafísico y una cantidad, que es precisamente por lo que no se puede medir directamente).
@Steve. Pero entonces el espacio tampoco se puede medir directamente...
@StéphaneRollandin, depende de lo que quieras decir. ¡Tal vez guárdelo para la próxima pregunta que haga!
@Steve: creo que el punto esencial sobre el concepto de tiempo es que se obtiene por abstracción de la observación de la coherencia de (la tasa de) diferentes procesos. En este sentido, no es tan diferente al concepto de longitud. Puedes usar una vara de medir, un pulgar, un metro, un pie, una braza para medir ciertas distancias, y siempre, dentro de los errores, tendrás la misma razón entre ellos. Así se forma el concepto abstracto de longitud.
@freecharly, tal vez eche un vistazo al último bloque que escribí en respuesta a Chris sobre el tema: chat.stackexchange.com/rooms/73393/… . Creo que me he explicado mucho más claramente allí.

La razón principal del cambio en el razonamiento ha sido una mejor comprensión/formulación del paso del tiempo.

Sí, los relojes miden el ritmo de una cosa u otra.

Las variables a las que se refiere en sentido clásico son variables específicas de diseño. Puedes diseñar dos péndulos casi idénticos pero con una fricción ligeramente diferente. Este es un problema de diseño.

En el caso de los relojes atómicos, los problemas de diseño no aparecen o, si lo hacen, son problemas de diseño muy diferentes y, una vez calibrados, la variación en estos diseños probablemente sea nula. En un reloj clásico (péndulo), dos relojes no solo pueden tener una fricción diferente al principio, sino que su fricción también puede variar de manera diferente con el tiempo. De ahí que las cuestiones clásicas se consideren verdaderas variables porque surgen del diseño y se pueden medir.

Ahora, en el caso de los relojes atómicos calibrados, se supone que se eliminan las variables específicas del diseño. Entonces, ¿cómo sigue variando la tasa de ticks? La tasa de ticks puede variar debido a situaciones que son independientes de los problemas de diseño. ¿Cómo puede detectar este cambio en la tasa de ticks, es un problema diferente?

Por ejemplo, hoy tienes un reloj atómico en un lugar fijo de la Tierra y funciona a un ritmo. Un mes más tarde, la propia Tierra se acerca, digamos, a un par de planetas vecinos debido a las circunstancias orbitales. Ahora el mismo reloj marcará a un ritmo diferente, pero no hay forma de que pueda detectar este cambio en términos de cambios/correcciones de diseño. Todos los relojes, clásicos/atómicos/buenos/malos sufrirán ese cambio en la tasa de tictac.

Ahí es donde la relatividad viene a ayudar. Habla del marco de referencia. Dos marcos de referencia diferentes tienen diferentes tasas de ticks. Entonces, cuando la tierra se acercó a otros planetas, en efecto, el marco de referencia (contando la gravedad) cambió y también lo hizo la tasa de tic. Para detectar este cambio, debe medirlo frente a la tasa de ticks en otro marco de referencia.

Dos relojes atómicos idénticos, uno colocado en el ecuador y otro en el polo a la misma altura desde el nivel del mar, marcarán a un ritmo diferente. Dos relojes atómicos idénticos, colocados a diferentes alturas desde el nivel de la vista en la misma línea vertical, marcarán a un ritmo diferente. Estas no son diferencias específicas del diseño, son diferencias específicas del marco y, mientras se encuentra en el mismo marco de referencia, no puede detectar estos cambios, haga lo que haga.

Entonces, en comparación con su ejemplo de temperatura, el cambio de temperatura cambia el marco de referencia. Si todos los procesos están perfectamente coordinados con la temperatura, no se pueden detectar estos cambios ya que la temperatura cambia asumiendo que no se puede medir la temperatura en sí y deducir que algo cambió.

El marco de referencia definido en la relatividad dice que todo está perfectamente coordinado en ese marco: "las leyes de la física son igualmente válidas en todos los marcos de referencia inerciales".

Los comentarios no son para una discusión extensa; esta conversación se ha movido a chat .

Hay muchos tipos diferentes de relojes que fabricamos, que (dejando de lado los efectos relativistas) se puede decir que funcionan con un tiempo más o menos constante. Un péndulo simple, por ejemplo, tiene un período de aproximadamente

T = 2 π yo gramo

Un péndulo, por supuesto, no es un gran reloj. Como ha dicho, su cronometraje "constante" puede verse frustrado de varias maneras. La mayoría de estos pueden compensarse de una forma u otra, pero al final no esperamos que un péndulo mantenga el tiempo perfecto. Lo que podemos hacer es descubrir todas estas fuentes de error y cuantificarlas; esto nos da una declaración como "este péndulo mantiene el tiempo con una precisión de hasta un segundo por año".

Ahora, dejando de lado los efectos relativistas, los relojes atómicos son el "estándar de oro" de los relojes constantes. El tiempo medido por dos relojes atómicos en el mismo marco de referencia, a la misma altitud, es extremadamente consistente. Ahora, estos relojes todavía no son perfectos, hay una ligera desviación debido a un error experimental. Sin embargo, en la década de 1990, esto era menos de un nanosegundo por día, como lo muestra este gráfico de incertidumbres de reloj, realizado por NIST:

ingrese la descripción de la imagen aquí

Ahora, a la relatividad. ¿Por qué creemos en la dilatación del tiempo? ¿Cómo sabemos que no son solo los relojes atómicos los que cambian de frecuencia?

En última instancia, esto se reduce al hecho de que la relatividad especial y general funcionan extremadamente bien. Recuerde que, en el momento en que se desarrolló la relatividad, no se habían medido los efectos de la dilatación del tiempo, ya que ningún reloj podía medir el tiempo con la precisión suficiente para detectar una desviación del diminuto efecto de la gravedad terrestre. La relatividad predijo que el tiempo se dilataría bajo ciertos efectos, y cuando nuestra tecnología de relojes se puso al día, nos dimos cuenta de que, de hecho, todo lo que podía considerarse un reloj actuaba exactamente como predijo la relatividad.

En última instancia, el tiempo es una construcción humana. Digamos que te subes a una nave espacial que viaja a una fracción significativa de la velocidad de la luz (digamos γ = 2 ) y volver a la tierra. Tu reloj atómico dice que estuviste viajando durante 10 años. Has envejecido 10 años. La experiencia se sintió como 10 años. Tu reloj dice que el viaje tomó 10 años. Su reloj radiactivo ha pasado por 10 años de vida media. En relatividad, solo dirías que el viaje tomó 10 años de tiempo adecuado. Todas estas serían definiciones razonables de tiempo, y todas concuerdan entre sí.

Por supuesto, puede hacer una nueva teoría donde el tiempo es absoluto, y cualquier medida de NIST-F2 es tiempo absoluto, y son solo todos los procesos que podrían interpretarse como un reloj que cambia de velocidad cuando no está en Boulder, Colorado ( que es donde está NIST-F2). Esto tiene algunos problemas:

  • Hace que sea literalmente imposible medir el "tiempo" en cualquier lugar fuera de Boulder, Colorado.
  • Es extremadamente geocéntrico. Lo cual es un poco insatisfactorio desde un punto de vista filosófico.
  • Estás abandonando la relatividad, lo que significa que ahora tienes que encontrar diferentes explicaciones para los muchos fenómenos aparentemente desconectados que explica la relatividad. Considere la precesión de la órbita de Mercurio, por ejemplo.
  • Si puede encontrar una teoría consistente (y tenga en cuenta que no estoy afirmando que pueda hacerlo) que explique todo lo que hace la relatividad, ciertamente será más complicado. ¿De qué se trata, pues?
Chris, agradezco esta respuesta, pero el hecho de que la relatividad funcione es irrelevante: nadie dijo nunca que no funcionaba o que estaba mal, así que no veo por qué necesitas ensayar ese punto con tanto detalle. Mi principal desafío está en cómo se describen sus efectos y las declaraciones engañosas sobre la constancia de la velocidad de los procesos atómicos. Además, te socavas a ti mismo, ya que si "el tiempo es una construcción humana", ¿cómo se puede decir que varía debido al entorno físico? Seguramente son las cosas medibles que no son constructos (como la "frecuencia del reloj") las que se considerarían variables.
@Steve En relatividad, la velocidad de estos procesos es constante: es el tiempo mismo el que cambia. "Qué es el tiempo" es una construcción humana, pero la definición de tiempo tal como se usa en la relatividad es una cosa física específica que varía. Puede cambiar el nombre del tiempo como "frecuencia de reloj inversa" si lo desea, e insistir en que "tiempo" es "frecuencia de reloj inversa medida en Colorado" si lo desea, pero es difícil ver lo que ha logrado excepto hacer que su teoría sea más geocéntrica .
@Steve Básicamente, bajo cualquier definición razonable de "tiempo", "velocidad de reloj inversa" es lo más parecido a "tiempo" que se puede medir localmente. Si la relatividad del tiempo le molesta, puede decir que el ICR de una persona es correcto y llamarlo "tiempo" o decir que "tiempo" es algo mágico e inconmensurable. Pero la relatividad del tiempo no molesta a los físicos, así que simplemente la llamamos tiempo.
No me preocupa especialmente la relatividad del tiempo, ¡más de lo que me preocupa la relatividad de las tasas de los procesos físicos! Parece que estás interpretando todo lo que digo como un rencor contra la relatividad; no lo es, es un rencor contra la falta de lógica. Mi pregunta original era simple: ¿qué justifica que hablemos de que "el tiempo se ralentiza" en lugar de hablar de "tasas de procesos que se ralentizan", y la respuesta que obtuve es nada en absoluto. Son afirmaciones equivalentes en relatividad y, en términos más generales, la última transmite el significado apropiado sobre la realidad física.
Todo se reduce a que si el tiempo no es relativo, entonces el tiempo tampoco es medible. Postular que el tiempo no es relativo también crea un marco de referencia preferido, que es una adición inútil a la teoría. Los relojes miden el tiempo (adecuado) por definición, y lo definimos de esta manera porque cualquier definición absoluta hace que el tiempo no se pueda medir (y, por lo tanto, sea un poco inútil)
No se requiere ninguna modificación a la teoría para reconocer que son las tasas las que son relativas. Son las tasas las que mide el reloj. El tiempo no se puede medir: solo se puede derivar numéricamente midiendo una tasa y luego afirmando una constante. Tal como lo tiene, el absolutismo del tiempo simplemente ha sido reemplazado por el absolutismo (la constancia) de las tasas de los procesos atómicos, por lo que no ha eliminado la afirmación de los absolutos de la teoría, simplemente los ha movido ( es decir, ha reemplazado la afirmación del tiempo absoluto, con la afirmación de las tasas atómicas absolutas). (1/4)
La razón por la que prefiero que el tiempo sea absoluto es porque no se puede medir directamente. Las tasas atómicas se pueden medir para ser variables. Bien puede ser la cantidad inconmensurable la que toma la tensión de ser lo absoluto. Ahora entiendo por qué el tiempo se usa como variable, porque el parámetro "tiempo" en el formalismo está sobrecargado de significado y se usa como un factor universal de corrección de velocidad en el formalismo, lo que hace que el formalismo de la física sea mucho más simple que introduciendo términos adicionales (que todos asumen el mismo valor) en cada tarifa. (2/4)
Además, más de un fenómeno físico requiere un factor de corrección de frecuencia y, nuevamente, debido a que se simplificó y se resumió en una corrección de tiempo, nunca está claro si el fenómeno físico es "aparente" (como el Doppler o la contracción de longitud). o uno "real" (como la dilatación del tiempo inducida por el movimiento, que realmente debería llamar "dilatación de la velocidad"). Es conveniente que el formalismo pueda hacer frente a todos estos efectos de una manera tan simple, ajustando el parámetro "tiempo" en todos los casos, pero ese parámetro temporal está ocultando una multitud de fenómenos físicos subyacentes diferentes. (3/4)
Finalmente, al volver al "tiempo absoluto" (pero no a las tasas absolutas), estamos más cerca de analizar qué es realmente el tiempo y qué papel juega en la teoría. Soy de la opinión de que es una cantidad abstracta derivada de la "dinámica espacial", es decir, no es una dimensión física independiente. No hay flujo físico de tiempo. Es una hipótesis abstracta para razonar sobre diferentes estados espaciales pasados ​​y futuros, y hacer que el "cambio" sea compatible con la geometría cuantitativa. Siendo abstracto y afísico, y siendo axiomático, no hay una buena razón por la que el tiempo no deba ser un "absoluto". (4/4)
...Y de nuevo, lo que este enfoque aclara es que cuando nada se mueve en el espacio, nada se mueve en el tiempo . El tiempo no pasa sin un cambio en la configuración espacial del sistema, precisamente debido a su papel de "dinámica espacial". Y el tiempo se repite si se repite toda la configuración espacial. Es decir, dos estados espaciales idénticos de un sistema, son indiferenciables en el tiempo. En la práctica, por supuesto, el universo entero no se repite (o no con frecuencia), ni se vuelve estático, por lo que los humanos están convencidos de que el tiempo siempre fluye, pero esa es otra historia.

Para responder a sus dos preguntas principales:

¿Qué miden los relojes?

Los relojes cuentan el número de veces que ocurre la misma secuencia continua de eventos.

Dado que la velocidad se mantiene constante, si la lectura del reloj varía, se puede atribuir a un cambio en "el paso del tiempo mismo", en lugar de un mero cambio en la velocidad del proceso que se está midiendo.

No deberíamos asumir esto. La secuencia continua de eventos que cuenta un reloj puede cambiar, haciendo que el reloj sea inexacto. Entonces, para dar cuenta de esto, los físicos crean una serie de relojes usando una secuencia diferente de eventos y luego comparan sus lecturas. Si la mayoría muestra la misma lectura dentro de un cierto error, entonces los físicos pueden estar cada vez más seguros de que la secuencia de eventos que cuenta cada reloj tampoco cambia, lo que hace que los relojes sean precisos.